MLT BOC study

Ace your homework & exams now with Quizwiz!

Bacterial colonies often emit distinct odors associated with foods. Which organism produces an odor that resembles fresh apples?

Alcaligenes faecalis (odorans)

A patient that is on a vegetarian diet will most likely have a(n):

Alkaline pH

Common crystals which can be found in ACID urine include:

Amorphous urates and calcium oxalate

All of the following laboratory tests are used to determine kidney function, EXCEPT?

Amylase

A patient who has a primarily vegetarian diet will most likely have:

An alkaline urine pH

Which of the following is a mechanism known to cause platelet satellitosis (satellitism)?

An antibody directed against the platelet membrane

A blood collection system that has a retractable needle malfunctions and causes an injury to a patient. Which of the following actions must be taken?

An incident report must be completed.

All of the following are cause for donor deferral EXCEPT?

An individual weighs 115 pounds at the time of donor screening.

Which of the following markers would you expect to find in the serum of a patient who has recently recovered from Hepatitis B infection?

Anti-HBs

A physician suspects his patient might have rheumatoid arthritis. Which of the following markers should be ordered?

Anti-IgG

If detected in antibody screen testing, which of the following antibodies is NOT considered clinically significant in prenatal patients?

Anti-Leb

Which of the following characteristics of hemolytic disease of the fetus and newborn (HDFN) is NOT different for ABO and Rh HDFN?

Antibody is IgG

Illustrated growing on the surface of the blood agar plate shown in the upper left image are small distinctly beta-hemolytic colonies suggestive of beta-hemolytic streptococci, particularly when specimens are obtained from patients with an upper respiratory infection. Gram-positive bacilli in branching and Chinese letter formations are observed on Gram stain as illustrated in the upper right image. Observe the reverse CAMP reaction in the lower image. From these observations, select the identification of this bacterial isolate.

Arcanobacterium haemolyticum

This suspicious form that is shown at both low (10X) and high dry (40X) power, was seen in a stool concentrate. The form measures 90 µm by 42 µm. This form is most likely a _________?

Ascaris lumbricoides egg

This suspicious form, shown below at both low (10X) and high dry (40X) power, measures 90 mm by 42 mm. It was seen in a stool sample. What is its identification

Ascaris lumbricoides egg

Exposure to all of the following chemicals may cause hemolytic anemia, EXCEPT?

Ascorbic acid

What is the most likely identification of a rapidly growing hyaline mold that began as a white colony and later develops a black "pepper" effect on the agar surface? As the colony aged it produced jet black, powdery colonies which then caused it to resemble a dematiaceous mold.

Aspergillus niger

Which Aspergillus species show biseriate with phialides covering the entire surface of a spherical vesicle with black conidia as microscopic morphological features?

Aspergillus niger

Choose the fungi that may be identified to a genus level, when the fungal structures that are seen in this photomicrograph are present in tissue sections

Aspergillus species

Which of the following substances may mask parasites in a stool sample?

Barium

Newborns who will develop Hemoglobin H disease may demonstrate up to 30% of which type of hemoglobin?

Bart's

A small, microaerophilic, curved, gram-negative rod which was isolated from a diarrhea stool specimen gave the following results: Grew best at 42 ºCOxidase positiveResistant to cephalosporinSusceptible to nalidixic acidHippurate hydrolysis positive What is the MOST probable identification of this organism?

Campylobacter jejuni

All of the following specimen collection methods should be used if urine cultures are also required, EXCEPT?

Routinely voided

What is the purpose of the BNP test?

Rules out acute heart failure in the emergency setting.

Anti-U antibodies can be produced by which of the following genotypes?

S-s-

Which of the following Streptococcus spp. exhibits a positive bile solubility test?

S. pneumoniae

A person who lived in the United Kingdom between the years of 1980 and 1996 for 3 or more months is indefinitely deferred from donating blood or blood products based on an increased risk of which of the following?

Creutzfeldt-Jakob disease (CJD) and/or variant Creutzfeldt-Jakob disease (vCJD)

Which component can be shipped with fresh frozen plasma (FFP)?

Cryoprecipitate

All of the following may be typically recovered in positive blood cultures, EXCEPT?

Cryptococcus neoformans

An encapsulated yeast that fails to produce a germ tube is urease positive and produces brown pigment on birdseed agar is found in a blood culture bottle from a patient with septicemia. What is the MOST likely identification?

Cryptococcus neoformans

A patient presents with cellulitis and culture growth small beta hemolytic colonies on 5% Sheep blood agar. The gram stain of the organism is included. What is the most likely cause of the infection?

Streptococcus pyogenes

Sodium thiosulfate is often added to selective culture media to:

Detect hydrogen sulfide Inhibit the growth of gram-positive bacteria ***Provide sulfur atoms for H2S positive isolates*** Serve as a pH buffer

The nitrite portion of the test strip can be used to do all of the following EXCEPT:

Detect metabolic by-products often seen in diabetic patients

In order to speciate a swarming Proteus organism found on an eosin methylene blue (EMB) agar plate, the best approach would be to:

Determine indole production and ornithine decarboxylase

All of the following are common support media used in electrophoresis techniques, EXCEPT?

Dextrose

Hemoglobin A1C (HbA1C) percent can be used as an assay to diagnose which of the following conditions?

Diabetes

Which of the following conditions is associated with a positive ketone test on the urine test strip?

Diabetes mellitus

The adult form that produces this 140 µm x 75 µm egg, typically inhabits the small intestine or bile ducts. What is the identification of this organism(s)?

Fasciolopsis/Fasciola species

Each of the following characteristics will differentiate between Listeria monocytogenes and Erysipelothrix rhusiopathiae except:

Fermentation of glucose

What is the principle of the Kleihauer-Betke test?

Fetal cells present in the mother's blood smear contain fetal hemoglobin (Hgb F). When treated with acid the fetal cells are resistant and will stain pink with the counter stain.

Parvovirus B19 is the only human pathogen in the family Parvoviridae. It is most known for its association with which of the following diseases?

Fifth disease

During which stage of a syphilis infection will a patient develop the painless chancre?

First stage

A routine serum protein electrophoresis with normal proteins results in _____ zones or bands.

Five

Trichomonas, Giardia, and Chilomastix belong to which one of the following parasitic classifications based on the type of motility?

Flagellates

All of the following are types of epithelial cells that line the genitourinary system EXCEPT:

Flattened cells

VDRL and RPR are examples of which of the following types of test methodologies?

Flocculation

Which of the following assays is more sensitive and specific for the diagnosis of Paroxysmal Nocturnal Hemoglobinuria (PNH)?

Flow cytometry for GPI-anchored proteins

Which of the following could cause a sickling event?

Fluctuations in temperature

All of the following characteristics of cerebrospinal fluid are considered abnormal EXCEPT:

Fluid is colorless

This photograph shows a positive reaction in each of the tubes with the tube on the left containing esculin medium and the tube on the right containing heart infusion broth with 6.5% sodium chloride. The bacterial species that characteristically produces these reactions is:

Enterococcus faecalis

All of the cells listed below function as a phagocyte, EXCEPT?

Eosinophil

This picture represents which type of leukocyte?

Eosinophil

Identify the arrowed cells in the image to the right from the Wright-Giemsa stained blood smear slide.

Eosinophilic Myelocyte

Charcot-Leyden crystals in stool may be associated with an immune response and are thought to be the breakdown products of:

Eosinophils

What is the identification of this cellular clump found in CSF? Note the presence of many similar-appearing nuclei without distinct lines of demarcation between cells.

Ependymal clump

A fungus culture from a toe-nail scraping demonstrates large, club-shaped macroconidia with no microconidia. The most likely identification is:

Epidermophyton floccosum

For which of these conditions or procedures there may be an increased number of megakaryocytes in the bone marrow, but a decreased number of circulating platelets?

Folic acid deficiency

Regarding hemoglobin synthesis, which of the following constitutes the alpha globin chain coding?

Four gene loci; two each on chromosome 16.

A normal hemoglobin molecule is comprised of the following:

Four heme and four globin chains

From the peripheral blood smears shown in the images to the right which image most likely corresponds with the blood smear that would be found from a patient with myelofibrosis?

Frame A

Which of the following is a correct pairing of cell type in the image and a disease state associated with that type of cell? All cells are from a Wright stained peripheral blood smear.

Frame C- primary myelofibrosis

Which bacterial genus contains fastidious, facultative intracellular, gram-negative bacilli that causes a zoonosis called tularemia?

Francisella

Illustrated in the upper image is a 4-day growth of a fungus colony on Sabouraud's dextrose agar. Note the cottony to woolly surface, and the distinctive lavender, pink pigmentation. The lower image is of a lactophenol cotton blue pigmented view of long, narrow multi-celled macro-conidia that are sickle-shaped. Based on these observations, select the genus identification of this isolate.

Fusarium

A small portion of HIV-infected individuals are termed 'elite controllers' due to only mildly depressed CD4 T cell counts and low viral loads. The presence of which of the following genetic factors is thought to contribute to this (low viral load and mildly depressed CD4 T cell counts)?

HLA-B57-01 and HLA-B27-05

Which of the following is also known as the Koch-Weeks bacillus?

Haemophilus aegyptius

A mother and young child arrive at the outpatient laboratory with a physician's order for a lead test on the child. The phlebotomist decides to perform a dermal puncture (fingerstick). What procedure is important to implement when a test for lead is collected by finger puncture that would NOT be necessary prior to an antecubital venipuncture?

Thoroughly washing patient's hands with soap and water.

Aerobic actinomycetes can cause different types of infections. All of the following are acceptable sites for the recovery of aerobic actinomycetes EXCEPT?

Throat

A 2-year-old girl, who had been treated with upper respiratory tract infection a few weeks ago, showed multiple bruises and had the following laboratory findings: Hgb = 13.5 g/dLRBC = 3.9 x 1012/LWBC = 8.0 x 109/LPlatelets = 5 x 109/LBone marrow M:E ratio = 3:1Megakaryocytes = increased These results are most suggestive of which condition?

Immune (idiopathic) thrombocytopenic purpura

Procedures that may enhance the recovery of Streptococcus pyogenes from throat cultures include:

Incorporation of sulfamethoxazole and trimethoprim in the culture media

Which of the following errors occurs in the post-analytical phase?

Incorrectly handled critical test value

Patients who develop severe sepsis or septic shock commonly have __________ plasma lactate values.

Increased

In an autoimmune disorder, conditions influencing the development of a disorder include the following factors with the exception of:

Increased discrimination of self from non-self antigens

Lymphocyte pleocytosis refers to a(n) _____________ in a CSF when compared to a normal sample.

Increased number of lymphocytes

The abnormalities that are seen on this slide are probably associated with which of the following conditions?

Increased red cell production

The results which would be MOST consistent with macroamylasemia are?

Increased serum amylase and normal to low urine amylase values

What is the correct method employers must notify employees regarding the results of formaldehyde monitoring?

Individually in writing or by posting the results in an appropriate location that is accessible to employees.

All of the following choices are acceptable specimens for anaerobic culture, EXCEPT?

Throat swab

The following test may be performed on all swarming Proteus species to differentiate between Proteus mirabilis and Proteus vulgaris:

Indole

Each of the following laboratory characteristics can be used to separate Bacteroides fragilis from the closely related Prevotella/Porphyromonas group of anaerobes EXCEPT:

Indole production

Elizabethkingia is the new genus name for the bacterium formerly called Chryseobacterium meningosepticum, an important agent of neonatal meningitis (now E. meningoseptica). What biochemical characteristic is unique for the family Flavobacteriaceae among the nonfermenters?

Indole production

The bacterial species shown on the blood agar plate illustrated in the photograph was recovered from a blood culture of a patient with fever. The most likely source of infection is which of the following?

Indwelling catheter

All of the following are associated with hereditary spherocytosis, EXCEPT?

Intravascular hemolysis

All of the following are reasons why molecular methods for the detection of methicillin-resistant Staphylococcus aureus (MRSA) have gained increasing attention, except:

Molecular methods are less expensive than culture methods.

Class II Major Histocompatibility Complex genes encode for:

Molecules that present antigen to CD4+ T cells

Which of the following best describes the benefits of the RPR or VDRL tests:

Monitoring course of treatment

After briefly circulating in the peripheral blood, which of the following cells becomes a tissue macrophage (or histiocyte) whose main function is phagocytosis?

Monocyte

Which of the following organisms is an aerobic, gram-negative diplococcus that does not ferment sugars and is DNAse positive?

Moraxella catarrhalis

Which of the following is the most likely identification of an acid fast bacilli recovered from an induced sputum with these culture findings? Slow growth at 37o C Niacin test = negative Pigmented photochromogenic Nitrate test = positive Catalase = positive

Mycobacterium kansasii

illustrated in this photograph are two Middlebrook 7H-11 plates incubated at 35°C, on which are growing colonies of an unknown Mycobacterium species. The plate on the left was incubated in the dark; the plate on the right had been exposed to light for 24 hours. The most likely identification is:

Mycobacterium kansasii

All of the following Mycobacterium species will cause tuberculosis, EXCEPT?

Mycobacterium leprae

Which of the following causative agents is most likely associated with the "swimming pool granualoma" skin lesions depicted in the attached photograph?

Mycobacterium marinum

The accumulation of niacin, as shown by the yellow reaction on the test strip shown in the right tube is one of the chief characteristics to identify Mycobacterium tuberculosis. Another rarely encountered Mycobacterium species that also accumulates niacin is:

Mycobacterium simiae

Illustrated in the top image are 10 day old rough, buff, gray-white colonies growing on Middlebrook 7H11 agar. These colonies are commonly recovered from expectorated or induced sputum specimens obtained from patients with signs and symptoms of pneumonia. The appearance of these colonies provide for a presumptive identification, supported by the positive yellow-pigmented niacin reaction seen in the reagent tube in the lower photograph. From these observations, select from the multiple choices the presumptive identification of this isolate.

Mycobacterium tuberculosis

Which of the following Mycobacterium species is positive for nitrate reduction?

Mycobacterium tuberculosis

Why is Mycobacterium tuberculosis not seen on gram stained smears?

Mycobacterium tuberculosis has a waxy coating on its surface which prevents gram stain penetration.

The course of multiple sclerosis can be monitored by measuring which of the following in cerebrospinal fluid (CSF) analytes?

Myelin basic protein (MBP)

Which of the following cells is seen in the photomicrograph?

Myeloblast

What is the primary oxygen-carrying protein found in muscle?

Myoglobin

Which of the following is the EARLIEST biochemical marker of myocardial infarction?

Myoglobin

In immunohematology, an antithetical relationship exists between M antigen and which of these antigens?

N antigen

Neisseria species can be traditionally differentiated by their fermentation of sugars. Identify the biochemical reaction that goes with the correct species of Neisseria given.

N. meningitidis = Glucose +, maltose +, lactose -

The nitrate reduction test is used to determine the ability of an organism to reduce nitrate. The Enterobacteriaceae group can reduce nitrate. Based on this test, what is the end product for which nitrate is reduced?

N2

What coenzyme does LD use when converting lactate to pyruvate?

NAD+

A Gram-negative diplococci grows on MTM (Modified Thayer Martin), is oxidase positive and superoxol positive. The carbohydrate utilization reaction seen in the QuadFerm system shown in the image provides an identification of which of the following organisms?

Neisseria gonorrhoeae

From the list of organisms below, identify the most likely species based on this image of a gram stain from a male urethral swab.

Neisseria gonorrhoeae

Which of the following BEST describes the relation of nephelometry to turbidimetry?

Nephelometry directly measures the amount of light scattered by particles in solution, and turbidimetry measures the decrease in incident-light intensity

A sputum sample from a patient with a pulmonary abscess is received in the laboratory. After several days of incubation the following findings are noted: Yellow, irregular colonies growing on a blood agar plate (BAP) under aerobic conditions. Acid-fast stain shows that the organisms are partially acid-fast. Which of the following organisms should be suspected?

Nocardia asteroides

The following organism is described as a branching Gram-positive bacillus that is partially acid-fast and able to hydrolyze urea, but unable to hydrolyze casein, xanthine, or tyrosine.

Nocardia asteroides

A maternal sample is collected 2 hours after delivery and a type and screen is performed. The screen is weakly positive and an anti-D is identified. Which of the following is the most likely cause of the positive antibody screen?

Passive anti-D from RhIG given at 28 weeks.

Which of the following statements regarding peak level in therapeutic drug monitoring is true?

Peak level is assessed by collecting a blood sample after a dose of the drug is given (at a specified time).

Regarding therapeutic drug monitoring, which of the following statements is correct when collecting a specimen for a peak level?

Peak level is tested on a blood sample that is collected after a dose of the drug is given (at a specified time).

What are apolipoproteins?

Proteins that are on the surface of lipoprotein molecules and bind enzymes or transport specific proteins and direct lipoproteins to their sites of metabolism.

The Western Blot for HIV-1 infection identification looks for HIV p24 and what other two protein bands to confirm HIV-1 infection?

gp41 or gp160

What is the main function of the hexose monophosphate shunt in red blood cells (RBC's)?

Provide NADPH and reduced glutathione to prevent oxidation of hemoglobin

Which of the following cardiovascular risk markers is a more sensitive version of a test that is used to assess inflammation?

hs-CRP

A stool specimen can be suspected of harboring Vibrio cholerae if it possesses which of the following characteristics?

if the stool is watery with a high pH and flecks of mucus.

Which of the following organisms is a lactose fermenter?

klebsiella oxytoca

A CLIA waived test requires that operators:

must follow manufacturer's instructions

What abnormality is present in this slide?

none

How is valproic acid (Depakote) administered?

oral preparation

Which of the following statements is TRUE with respect to uncompensated metabolic alkalosis?

pH will be elevated without an elevation of pCO2

Through the use of molecular assays, the Mycobacterium tuberculosis (MTB) complex can be detected directly from respiratory specimens without being concentrated. In addition, due to rifampin resistance strains of MTB complex occurring, a genetic marker can be detected to identify rifampin resistance. Which of the following genetic markers is linked to MTB complex rifampin resistance?

rpoB gene

The following is true of the disk diffusion method of measuring antimicrobial sensitivity:

Provides only an interpretation value of sensitive, intermediate, or resistant correlated to MIC values

The primary site of reabsorption of glomerular filtrate is the:

Proximal convoluted tubule

In an adult, hematopoiesis occurs in which of the following listed sites?

Proximal ends of long bones

Which of the following is oxidase-positive?

Pseudomonas aeruginosa

Myoglobin concentration testing is routinely used for all of the following conditions, EXCEPT?

Pulmonary edema

Some strains of Group F Strep can carry the group A antigen, potentially leading to a false positive identification of group A, Streptococcus pyogenes. A spot test that is helpful in differentiating these two species and thus preventing a false report is:

Pyrrolidonyl-b-naphthylamide (PYR)

Matrix-assisted laser desorption-ionization time-of-flight (MALDI-TOF) is used for identification of aerobic bacterial isolates on a urine culture bench. Upon placing the target into the MALDI-TOF instrument, no peaks were found on the quality control wells, but the patient isolates gave good identifications. What caused this issue?

Quality control organism was not added to the target wells

A new lot of urinalysis chemical reagent strips is received in the laboratory. When should quality control (QC) be performed on the new strips?

Quality control should be performed on the new lot of chemical strips as soon as they are needed for testing in the laboratory.

What is his MOST likely genotype for a white male who has the following Rh antigens C,c,D,E,e?

R1R2

Given the following commonly used nomenclature systems, which one of the Rh genotypes listed below is heterozygous for the C antigen? Weiner HaplotypeFisher-Race HaplotypeR0DceR1DCeR2DcERzDCErdcer'dCer"dcErydCE

R1r

MCV is calculated using which of the following parameters?

RBC and Hct

What is the common name for Hymenolepis diminuta?

Rat tapeworm

The kidneys perform all of the following functions EXCEPT:

Reabsorption of waste products

An automated hematology counter flagged the white blood cell count. Upon reviewing the peripheral blood smear, the technologist viewed many cells that appeared similar to those in this image. What should the technologist report?

Reactive/atypical lymphocytes

All of the following statements are true for the urinalysis reagent strip procedure, EXCEPT?

Reagent strip should be immersed in centrifuged urine specimens only.

The disappearance of HBsAg and HBeAg, the persistence of Total anti-HBc, the appearance of anti-HBs, and often the presence of anti-HBe indicates what type of Hepatitis infection?

Recovery phase of HBV infection

The element that is indicated by the arrow is a:

Red blood cell cast

False negative results may occur with indirect antiglobulin tests as a result of all of the following EXCEPT:

Red blood cells have a positive DAT

What characteristic is common to all of these conditions: hereditary spherocytosis, hereditary elliptocytosis, hereditary stomatocytosis, and paroxysmal nocturnal hemoglobinuria?

Red cell membrane defects

Which one of the following hematological determinations is MOST reliable for evaluating bone marrow erythropoietic activity?

Reticulocyte count

Which of the following statements characterize a reticulocyte?

Reticulum visible with a supravital stain, such as new methylene blue

The ideal stool sample for parasitic examination is one that is freshly collected and submitted to the laboratory at:

Room temperature (approximately 22° C)

A blood culture yielded an organism with the following reactions: TSIA: K/A, gas (+), H2S (+) Citrate: Positive Indole: Negative Lysine: Positive Ornithine: Positive Motility: Positive Phenylalanine: Negative ONPG: Negative Urease: Negative What is the most likely identification of this organism?

salmonella

A medical laboratory scientist is working with a student on the teaching microscope. The student is using a microscope that has a micrometer for the measurement of microscopic items. A normal red cell distribution curve for a patient is shown on the right. What should the range of red cell diameters be when the student measures the red cells with a micrometer?

6-8 microns

Using an exclusion protocol that requires only one homozygous cell to exclude antibodies, three antibodies have not been excluded in the panel shown below. Two of the antibodies are anti-E and anti-K, what is the third antibody that is possible (has not been excluded)?

Anti-Jkb

A cerebrospinal fluid sample from a possible meningitis patient demonstrated a round, budding yeast of variable size on Gram stain. The specimen was cultured to routine fungal media without cycloheximide. The organism rapidly produced urease and demonstrated brown pigmented colonies on niger seed agar. The organism identification is:

Cryptococcus neoformans

Which of the following would be the BEST data source to identify opportunities to improve timeliness?

Outliers in turnaround time studies

A member of the Enterobacteriaceae is inoculated into various biochemical media which are shown in the image. Shown from left to right are triple sugar iron agar (TSIA), lysine iron agar, urea agar, citrate agar, phenylalanine deaminase agar, and motility indole ornithine agar. This organism is most likely:

Providencia spp.

All of the following are true in regards to performing quality control (QC) on urine chemical reagent strips, EXCEPT:

QC is performed only when patient results are questioned by the health care provider

A patient with an infectious mononucleosis infection presents in the emergency room. Physicians order a spinal tap, which is immediately sent to the laboratory for review. What is the cell in the image below from this patient's cerebrospinal fluid sample?

Reactive Lymphocyte

How is the Mean Cell Volume calculated?

(Hematocrit %/ Red blood cells) X 10

What is the half-life of immunosuppressive drugs cyclosporine and tacrolimus?

10-12 hours

What is the minimum hemoglobin concentration required for autologous blood donation?

11 g/dL

What is the normal ratio of erythroid to myeloid cells found in the normal bone marrow?

1:3

Whole blood if composed of approximately:

45% formed elements, 55% plasma

How do you make up 275 mL of a 4.6 M solution of HCl (gmw=36.46 g/mol)?

46.1 g HCl

Adult (normal) Hemoglobin is made up of the following composition:

>95% HbA, <3.5% HbA2, <1-2% HbF

What is a hemiacetal?

A compound with both a carbonyl and alcohol group.

The ketone component that is measured by the nitroprusside reaction is:

Acetoacetic acid

A CSF specimen is received from a patient with suspected meningitis. Positive Gram stain, along with the results shown here suggest what type of condition? Elevated WBC count Neutrophils predominant Marked protein elevation Marked glucose decrease Lactate level = 37 mg/dL

Bacterial meningitis

The predominant cells seen on the CSF smear in this illustration are indicative of:

Bacterial meningitis

All of the following organisms are aerobic organisms, EXCEPT?

Bacteroides fragilis

Identify the nucleated blood cell in the image.

Band neutrophil

Which of the following hemoglobins is replaced by hemoglobin H (HbH) after birth in individuals with HbH disease?

Bart's

Identify the illustrated leukocyte.

Basophil

Identify the nucleated blood cell:

Basophil

This image represents a Wright-Giemsa stained peripheral blood smear. What are the inclusions that are present in the red blood cell indicated by the arrow?

Basophilic stippling

Cyclosporine and tacrolimus (both immunosuppressive drugs) have multiple therapeutic ranges, why?

Because both drugs have ranges dependent on the organ transplanted and the time after transplantation.

When should a pipette be wiped off?

Before lowering the meniscus to the calibration mark

Which disease/condition is associated with a deficiency in thiamine?

Beriberi

The immunoassay procedure for serum hCG utilizes antisera against which subunit of hCG?

Beta

A patient who was seen in the ED has symptoms consistent with cellulitis. A gram stain from the patient's blood culture revealed gram-positive cocci in pairs and short chains. Blood cultures grew an organism that was susceptible to a 0.04 unit disk of bacitracin. Other characteristics one would expect to see with this bacterial isolate include:

Beta hemolysis, negative catalase, negative CAMP, growth in 6.5% salt

A beta-hemolytic Streptococcus was isolated from a throat culture. It gave the following biochemical reactions: Bacitracin - Susceptible Bile esculin - Negative 6.5% NaCl - No growth CAMP - Negative SXT - Resistant What is the presumptive identification?

Beta hemolytic Streptococcus, Group A

"Xanthochromic" CSF means that the color of the CSF can be any of the following EXCEPT:

Blue

All of the following are Lancefield carbohydrate antigens found on beta-hemolytic Streptococcus species. All of the following antigens are found on organisms from the Streptococcus anginosus group EXCEPT?

B

Red Cells Tested With Known Antisera Serum Tested With Known Red Cells Interpretation of ABO Group Anti-A Anti-B A1 Cells B Cells 0 4+ 4+ 0 ? Using the information provided above, what is the patient's correct ABO group?

B

Which of the following leukocytes is most directly associated with antibody production?

B cells

Which of the following D variants has the best likelihood to receive D-positive RBCs without any adverse effects?

C in Trans to RHD

The microtubules that function as mitotic whips or sensory devices on the surface of the cell or serve as tracks for the transport of materials down long neuronal axons are called:

Cilia

Which of the following smear techniques has the advantage of providing the best WBC distribution when performed correctly?

Coverslip technique

An increased serum level of which of the following analytes is MOST commonly associated with decreased glomerular filtration?

Creatinine

Which of the following analytes would be increased due to delay in centrifugation?

Creatinine

Risk management as it applies to the clinical laboratory is most accurately defined in which of the following ways?

Creation of policies and procedures and the implementation of practices that focus on prevention of future events that could result in negative patient outcomes.

The measurement of sodium and chloride in the sweat is the most useful test for the diagnosis of what condition/disease?

Cystic fibrosis

Stenotrophomonas maltophilia and Burkholderia cepacia share each of the following characteristics EXCEPT:

Cytochrome oxidase activity

Which of the following phrases best describes a segmented neutrophil?

Cytoplasm appears pink-purple due to small specific granules.

Illustrated in this photograph are colonies growing on the surface of a chocolate agar plate. The colonies were recovered from a subcutaneous infection of the forearm and had a bleach odor. The most likely identification is:

Eikenella corrodens

Electrophoretic separation of hemoglobin fundamentally relies on:

Electrical charge differences of molecules

Which one of the following statements about lipoproteins is FALSE:

Elevated LDL causes a decreased risk of atherosclerosis

Motility is a key initial assessment in the presumptive identification of the nonfermentative gram-negative bacilli. All of the following organisms are motile EXCEPT?

Elizabethkingia meningoseptica

What is the abnormal RBCs shape seen in this illustration?

Elliptocytes

Which phrase describes enrichment media?

Encourages the growth of specific types of organisms.

Increases in blood ammonia levels would be expected in which of the following conditions:

End-stage cirrhosis

A 12-year-old female visited her family doctor for her yearly back-to-school check-up. She was in good health and asymptomatic at the time of the examination. Due to an increased incidence of parasitic infections in the area, the doctor ordered a stool for parasite examination. Multiple suspicious forms measuring approximately 9 µm as seen in this image were present. Which of the following is the most likely identification of these forms?

Endolimax nana

Before disposing of a needle in the sharps containers, what should you do?

Engage the needle safety device

What parasite's trophozoite form ingests white blood cells, measures approximately 5 - 15 µm, and typically resides in the mouth?

Entamoeba gingivalis

Shown in this photograph are three tubes of Moeller decarboxylase broths, containing from left to right, the amino acids lysine, arginine, and ornithine respectively. A red-blue reaction indicates decarboxylation of the amino acid; yellow is negative. From the bacterial species listed, these reactions are most likely caused by:

Enterobacter aerogenes

What nematode egg is characteristically flattened on one side?

Enterobius vermicularis

Which one of these Gram-positive species has a high incidence of vancomycin resistance?

Enterococcus faecium

Identify the nucleated blood cell.

Eosinophil

identify the urine sediment

Fatty cast

What protective methods should be used when working with organic solvents?

Fume hood

According to the American Diabetes Association recommendations, which of the following statements is TRUE with regards to the following findings? Fasting glucose = 130 mg/dL 2-hour post prandial glucose = 210 mg/dL

The patient may be diagnosed as having diabetes mellitus

A physician is evaluating a 45-year-old obese male for diabetes and orders a plasma glucose test at the time of evaluation and an HbA1C one week later. The patient has a family history of diabetes and currently exhibits symptoms of diabetes. What would be the best course of action if these are the blood glucose test results? Casual Plasma Glucose: 208 mg/dL HbA1C one week later: 7.2%

The patient meets the criteria for diagnosis of diabetes.

What conclusion can be made if a person has a cardiovascular risk marker that is abnormal?

The person has an increased probability of developing cardiovascular disease.

Which of the following factors would least affect an orally administered drug's absorption and/or distribution in the body?

The position of the patient when the drug is taken (sitting vs. standing)

Which of the following statements is TRUE for specific gravity measured by the reagent strip method?

The reagent strip method is based on the relationship between ionic concentration and specific gravity.

Avidity is best described by which of the following statements?

The strength with which multivalent antigens and antibodies bind

Contact with infected cat feces is associated with the transmission of:

Toxoplasma gondii

Which of the following reagent strip tests is based on the Ehrlich-aldehyde reaction?

Urobilinogen

Illustrated in this photograph is the surface of a thiosulfate citrate bile sucrose (TCBS) agar plate which are growing colonies of an unknown bacterial species from a diarrheal stool specimen. The most likely identification is:

Vibrio cholerae

Which of the following organisms is known to be a strict halophilic organism?

Vibrio vulnificus

The most significant class of microbial targets for Natural Killer (NK) cells is:

Viruses

Which of the following analytes is light sensitive?

Vitamin B12

Nephelometry involves the measurement of:

light scatter

Carbapenemases are produced by various organisms such as Enterobacteriaceae, Pseudomonas aeruginosa, and Acinetobacter species. Detecting the presence of carbapenemase activity in these organisms is important to prevent the spreading of resistant organisms. Which of the following methods is used to detect the presence of carbapenemase-producing organisms?

mCIM

Which of the following terms is used to express the average of a series of numbers or values:

mean

MIC is an acronym for:

minimum inhibitory concentration

Which of the following can lower the amount of current needed to cause electricity-induced injury?

wet skin

Which of the following genotypes cause beta thalassemia minor?

ß0/ß

All of the following are considered to be important protein regulators of iron metabolism, EXCEPT?

Hemosiderin

Which of the following additives should be used for the collection of a sample for blood gas analysis?

Heparin

What is the MOST frequent disease complication of blood transfusions?

Hepatitis

Which of the following Hepatitis viruses is (are) transmitted through the fecal oral route and may be implicated in infections transmitted through contaminated food and water?

Hepatitis A

Which of the following is a DNA virus?

Hepatitis B virus

Increased concentration of alpha-fetoprotein (AFP) in adults are most characteristically associated with:

Hepatocellular carcinoma

A peripheral smear with red blood cells photographed in a typical field was submitted for review. All of the following conditions are most likely associated with the red blood cell population found here, EXCEPT:

Hereditary Hemochromatosis

Which hematologic condition could be associated with the findings shown in these images?

Hereditary spherocytosis

Which genetic description will NOT contain Bart's hemoglobin at birth ?

Heterozygous alpha thalassemia-2

Within the genus Campylobacter, the laboratory test most helpful in selectively identifying Campylobacter jejuni is:

Hippurate hydrolysis

The granules that basophils contain are composed of:

Histamine

The image on the right is a representative field from a peripheral blood smear. What characteristics would describe the red cell distribution curve that you would expect to see?

Histogram with a shift to the left

From the fungi listed below, which is considered a true pathogen that also infects immunocompromised patients?

Histoplasma capsulatum

The morphologic structures found in select tapeworms that are typically responsible for attaching to the human intestinal wall are called:

Hooklets

In examining a Wright's stained blood smear of a patient who has had a splenectomy, one would most likely also find:

Howell-Jolly bodies

Which of the erythrocyte inclusions listed below would appear on a Wright-stained peripheral blood smear as round, smooth, almost pyknotic, dark-purple staining bodies ranging in size from 0.5 to 1.0 micron in diameter and usually occurring singly in erythrocytes?

Howell-Jolly bodies

An occupation or avocation that should be avoided to prevent infection with Francisella tularensis is:

Hunting and skinning small game

Which of the following casts might be found in the urine of a healthy individual after strenuous exercise?

Hyaline casts

The primary mechanism responsible for glomerular filtration is:

Hydrostatic differential in glomerular tufts

Which of the following terms could be used to describe the red blood cells in this field?

Hypochromic

The automated cell count reported an MCV of 75 fL for the blood specimen from which this blood smear was made. Which of the following terms best describes the cells in this field?

Hypochromic/microcytic

What morphological change is present in this image?

Hyposegmentation of the nucleus

What is the inside portion of the needle tube called?

Lumen

The white blood cell that can vary MOST in size and shape on a Wright-stained smear is:

Lymphocyte

Which of the following white blood cells can appear atypical or reactive on a Wright-stained smear?

Lymphocyte

All of the following white blood cells would produce a positive leukocyte esterase test on the urine chemical reagent strip, EXCEPT?

Lymphocytes

What are the smallest nucleated cells seen in normal peripheral blood?

Lymphocytes

A patient suffering from typical leukemic symptoms presents in the emergency room. Physicians order a spinal tap after noticing possible central nervous system involvement. What are the cells seen in the spinal fluid cytospin preparation?

Lymphocytes and Blasts

Which of the following statements is TRUE regarding lymphocyte cytoplasmic granules?

Lymphocytes may contain a few azurophilic granules in the cytoplasm.

The process of freeze-drying of bacteria is called:

Lyophilization

A technologist is having trouble differentiating between red blood cells, oil droplets, and yeast cells on a urine microscopy. Acetic acid should be added to the sediment to:

Lyse the red blood cells

All of the following activities are associated with platelets EXCEPT:

Lysis

Which of the following sites is used most often for CSF collection?

L3-L4

From the choices listed below, which organism is classified as an aerotolerant anaerobe?

Lactobacillus species

ABO blood groups were discovered by:

Landsteiner

Which of the following is characteristic of Alder-Reilly anomaly?

Large, darkly staining cytoplasmic granules composed of partially digested mucopolysaccharides

What is the name of the structure formed when introns form a loop on themselves after being cut from the RNA?

Lariat

Which of the following disorders or diseases is associated with coarse basophilic stippling?

Lead poisoning

Which Lewis antigen(s) would be detected when phenotyping the red cells of an adult who has the Le, Se, and H genes?

Leb

All of the following are causes of hypernatremia EXCEPT:

Low aldosterone production

Which of the following RBC indices is expressed in femtoliters?

MCV

Antigen processing is primarily accomplished by what type of cells?

Macrophages

When performing sequence-based analysis to identify a bacterial isolate, all of the following must occur to ensure accurate results are obtained EXCEPT?

Mixed cultures can be used as specific sequence is being identified

Out of the choices below, which set of cells is arranged from least mature to most mature?

Stem Cell, Rubriblast, Prorubricyte, Rubricyte, Metarubricyte, Reticulocyte, Erythrocyte

Which of these is arranged from least mature to most mature?

Stem cell, lymphoblast, prolymphocyte, lymphocyte

What are the abnormal red blood cells (RBCs) indicated by the arrows that may be suggestive of an individual possessing the Rh null phenotype?

Stomatocytes

The identification of Neisseria species is most often confirmed by observing the specific carbohydrate utilization profile (glucose/dextrose; maltose; lactose; sucrose; yellow is a positive reaction). Based on the carbohydrate reactions as observed in the image, identify the organism: Image courtesy of the CDC

Neisseria meningitidis

Gram-negative cocci that occur in pairs with their adjacent sides flattened, giving them a "coffee bean" appearance (indicated by the arrows in the image), are typical of which of the following bacteria?

Neisseria species

Enterobius vermicularis (pinworm) is classified as which of the following?

Nematode

A chemotactic movement is demonstrated by:

Neutrophils

An individual with group AB blood does not have which of the following antigens present on their red blood cells?

O

Which one of the ABO groups listed below has the MOST H antigen?

O

What are the possible ABO genotypes of offspring from parents whose genotypes are OO and AB:

OA and OB

If an individual has blood type O, which of the following are possible genotypes?

OO only

What are the possible ABO genotypes of offspring of parents whose genotype is AO and BO?

OO, AO, BO, AB

The statement, "The student will perform direct antiglobulin tests with 100% accuracy," is an example of a(n)?

Objective

A phlebotomist is going to perform an arterial blood gas collection. This can be a painful procedure. All of the following steps are recommended to be taken by the phlebotomist in order to ease the patient's anxiety, EXCEPT?

Obtain the specimen as quickly as possible after verifying the patient identity.

To determine if xanthochromia is present in a cerebrospinal fluid sample, the sample must be centrifuged within:

One hour

Which of the following tests should be incubated at 37º C in a 5-10% CO2 atmosphere?

Optochin disk test

A tool used to show chain of command by defining responsibility and authority is a/an:

Organizational chart

All of the following are methods employed for measuring the specific gravity of urine, EXCEPT?

Osmolality

The cells in this image are from a bone marrow smear. Identify the cells present in this image:

Osteoblast

The large cell shown in the illustration to the right is occasionally seen in the bone marrow and can be mistaken for a plasma cell. What is this cell?

Osteoblast

Which of the following is considered a cause of hypomagnesemia (decreased levels of magnesium in the blood)?

Pancreatitis

A few dark blue staining granular inclusions located near the periphery of an erythrocyte are most likely:

Pappenheimer bodies

Degenerated erythrocyte cytoplasmic organelles that contain iron are called:

Pappenheimer bodies

The red blood cell inclusions noted in the Wright stained peripheral smear to the right are called:

Pappenheimer bodies

Which of the following red blood cell inclusions, marked by arrows, is seen on this peripheral blood smear?

Pappenheimer bodies

A 38-year-old male presented to the E.R. complaining of severe cough, chest pain, shortness of breath, and fatigue. Microscopic examination of a bloody sputum specimen revealed this suspicious form. The patient is most likely suffering from:

Paragonimus westermani

identify the cells at the arrows

Polychromatophilic erythrocytes

Which of the following is a nonfermenter?

Pseudomonas aeruginosa

Which of the following would not qualify for continuing education credit through the ASCP credential maintenance program?

Reading laboratory-related journal articles

Which method remains the "gold standard" for ANA detection?

Slide-based immunofluorescent assay (IFA)

Which immune elements are involved in a POSITIVE TB skin test?

T cells and macrophages

All of the following methods are used to detect and differentiate white blood cells in most hematology analyzers, EXCEPT?

Lysis of white cells with subsequent conversion to cyanmethemoglobin followed by spectrophotometric determination

All of the following are virulence factors utilized by Streptococcus pneumoniae to evade host defenses when causing meningitis, EXCEPT:

M protein

When assessing a bone marrow aspirate smear of an adult, a normal M/E ratio (myeloid to erythroid cells) would be?

M:E = 2.5:1

In patients with Sickle cell disease, upon sickling what laboratory test will see an increase?

MCHC

Which of the following set of results would be consistent for a patient with the following findings: macrocytosis, anemia, leukopenia, and thrombocytopenia?

MCV 115 fL; Hgb 7.5 g/dL; WBC 2.5 x 103/µL; Platelets 75,000/µL

The education coordinator for the hospital is required to develop competency assessment activities for the POCT providers in the emergency department. What document may serve as a guideline for specific steps to be included in the assessments?

Manufacturer's instructions

What information can you find on the label affixed to the primary container of a hazardous chemical?

Manufacturer's name and contact information

Myoglobinuria is MOST likely to be noted in urine specimens from patients with which of the following disorders?

Massive muscle trauma

Which of the marrow cells from the list below has highly granular basophilic cytoplasm but does not show nuclear lobules?

Mast cell

How many doses of RhIG should be administered if the calculated amount of fetal whole blood volume in maternal circulation was 28 mL?

2 doses

When should acute and convalescent blood specimens be collected from a patient for the detection of antibody concentration related to a specific infectious disease?

2 weeks apart

What is the half-life for aminoglycoside antibiotics in the blood?

2-3 hours

In a moderately bloody specimen, 250 cells are counted in 1 primary square. The dilution is 1:100. What is the count/µL?

2.5 x 10^5

Approximately how many out of 1,000,000 Caucasians will have the following phenotype?Group 0, K+, Jk(a+)

30,000

What is the half-life of digoxin in plasma?

38 hours

The accepted interval between whole blood donations is:

8 weeks

What blood alcohol concentration has been established as the statutory limit for the operation of a motor vehicle in the United States?

80 mg/dL

Which of the following represents the approximate percentage of the Caucasian population that is Rh-positive?

85%

What is the approximate volume of spinal fluid in an adult?

90-150 mL

From the IgG molecule illustration, which region is the heavy chain?

A

Which of the following statements regarding the L/S ratio is TRUE?

A ratio of 2:1 or greater usually indicates adequate pulmonary surfactant to prevent respiratory distress syndrome (RDS)

Which of the following statements is true regarding cardiovascular disease and adipose tissue?

A state of abnormal lipids and lipid levels found in metabolic syndrome contribute to the development of cardiovascular disease

A false-negative reaction while performing the DAT technique may be the result of:

AHG addition delayed for 40 or more minutes

A hepatic function panel A consists of which of the following tests?

AST, ALT, Alkaline Phosphatase, Total Protein, Albumin, Total Bilirubin, Direct Bilirubin

All of the following conditions are consistent with this peripheral blood picture, EXCEPT?

Abetalipoproteinemia

When collecting blood from the antecubital area using a syringe system, what should the insertion angle be for the needle?

About 30 degrees

Which of the following best describes the size and staining characteristics of yeast cells on a Gram stain?

About the same size as red blood cells and stain purplish-blue.

All of the following are collected from sterile sites, EXCEPT:

Abscess fluid

What is the predominant abnormal RBC shape seen in this illustration?

Acanthocyte

What are the abnormal red blood cells that are indicated by the arrows in this image?

Acanthocytes

After birth, which of the following is the biggest concern for a newborn infant suffering from hemolytic disease of the fetus and newborn?

Accumulation of bilirubin

Which of the following statements regarding a biomarker with high sensitivity is TRUE?

Accurately identifies the presence of disease and has few false negatives.

ß-hydroxybutyric acid is formed as the result of an accumulation of which of the following?

Acetyl-CoA

When identifying Mycobacterium spp, which of the following methods has the lowest sensitivity?

Acid Fast staining

Which group of conditions increases the risk of HbS polymerization?

Acid pH, dehydration, increased level of 2,3-DPG

Using an automated cell counter analyzer, an increased Red Cell Distribution Width (RDW) should correlate with which of the following?

Anisocytosis

Which of the following is a general term describing variation in red cell size?

Anisocytosis

Which of the following arthropods is an important part of the life cycle of Plasmodium species

Anopheles

If a patient is determined to have type B blood, what antibody would be found in his/her serum?

Anti-A

Lectin from the seeds of Dolichos biflorus is a source of which of the following?

Anti-A1

If a patient is determined to have type A blood, what antibody would be found in his/her serum?

Anti-B

A technologist performing an antibody identification in the blood bank has recorded the following results found in the worksheet below. Which antibody(ies) listed below cannot be ruled out?

Anti-C, anti-K, anti-E

If an Rh negative patient is administered a unit of R1R1 packed red cells, which one of the following antibodies would be most likely to develop:

Anti-D

If an R1r patient received R2R2 blood, which of these antibodies could be produced?

Anti-E

Which antibody is present in the blood of an individual with the Bombay phenotype that will cause agglutination with any non-Bombay individual's blood?

Anti-H

Which of the following is necessary to test for in order to detect patients in the "window" period of a hepatitis B infection?

Anti-HBc

All of the following antibodies are detected at immediate spin EXCEPT?

Anti-Jka

Which risk marker has both positive and negative cardiovascular risk integrated into its measurement?

Apo B-100/Apo A-1

Which of the following statements is true concerning apolipoproteins?

Apolipoproteins are essential for lipid metabolism.

Which enzyme is responsible for the rate-limiting step of steroid hormone synthesis?

CYP450

Trypanosoma cruzi is the etiologic agent of:

Chagas' disease

At a minimum, the following class of biological safety cabinet (BSC) must be used when manipulating potential bioterrorism agents and if aerosols are likely:

Class II BSC

All of the following are characteristic of an exudate effusion, EXCEPT:

Clear appearance

Eikenella corrodens is associated with which of the following?

Clenched fist infection

Which of the following clinical laboratory departments performs the most tests?

Clinical Chemistry

Which of the following organisms is gram-positive?

Clostridioides difficile

The image in the upper right indicates the mold form of which dimorphic fungus?

Coccidioides immitis

Which of the following fungal infections is endemic in the Southwestern United States desert regions?

Coccidioidomycosis

A woman with the blood type AB will show antigens for both the A and B blood groups. This is an example of what type of inheritance?

Codominance

In the ABO blood group system, the A antigen is inherited in what relation to the B antigen?

Codominant

An increase in carcinoembryonic antigen (CEA) levels is most closely associated with which of the following organs?

Colon

Protozoal parasites that typically do not produce disease in humans are referred to as being:

Commensal

_______________ is predominantly associated with skin and soft tissue infections (SSTIs), such as abscesses, cellulitis, folliculitis and impetigo.

Community-acquired MRSA

A 40-year-old man patient presents with non-specific physical complaints which include abdominal pain. Based on symptoms and family history the physician suspects Hereditary Hemochromatosis. Which of the following laboratory tests would be the LEAST diagnostic?

Complete blood count

The renal threshold is best described as:

Concentration at which a substance in the blood spills into the urine

In which of the following cases should Rh Immune Globulin (RhIG) be given?

Ectopic pregnancy

Identify the nucleated blood cell:

Eosinophil

The organism pictured below was a Gram-positive rod isolated from a sore on the hand of a fish market employee. It is shown on triple sugar iron agar. The organism is also catalase-negative and non-spore-forming. The most likely identification is:

Erysipelothrix rhusiopathiae

Which hormone is produced by the kidney and influences erythrocyte production?

Erythropoietin

The term used to describe patients with absence of Rh antigens is:

Rhnull

What is the most likely identification of a pink yeast isolate, recovered from respiratory secretions of a patient with AIDS, that produced the following results? Germ tube - negative Urease - positive Blastospores - positive Arthrospores - negative

Rhodotorula spp.

The two most commonly used antibiotics for therapy against uncomplicated tuberculosis are:

Rifampin and Isoniazid

This image demonstrates which of the following forms of Plasmodium species:

Ring forms

All of the following are present in normal bone marrow EXCEPT?

Ringed sideroblast

Which of the following would be considered most significant as it relates to serological testing:

Rise of antibody titers is diagnostic

Which physician is known for their discovery of Bacillus anthracis and Mycobacterium tuberculosis?

Robert Koch

The RBCs indicated by the arrows in this image are the result of:

Rouleaux formation

Which of the following describes normal lymphocyte characteristics?

Round to oval nucleus comprising 90% of the cell with a blue cytoplasm

Which one of the following specimen types is suitable for the recovery of Entamoeba histolytica?

Stool

A tech is performing an antibody screen. The reaction is negative when reading at the AHG (anti-human globulin) phase, so the tech then adds Coombs' control cells. Agglutination is now observed. What does this indicate?

The negative reaction at AHG phase is valid

The CDC states that, according to the current HIV testing algorithm, specimens with reactive initial antigen/antibody combination immunoassay should be tested with an FDA-approved antibody immunoassay that differentiates between HIV-1 antibodies from HIV-2 antibodies. After the differentiation assay:

The nonreactive and indeterminate results should be followed by nucleic acid testing.

Which of the following inclusions are frequently seen on the same peripheral blood smear along with toxic granulation?

Vacuoles

Which of the following parts of the body synthesizes and secretes BNP?

heart

Which of the following is TRUE of a fully compensated metabolic acidosis?

pH will be normal with a decrease in pCO2

The condition most likely to predispose a person to septicemia by viridans streptococci is:

poor oral hygiene (The oropharynx is thought to be the source in most cases of viridans streptococcus with organisms entering the tissue during dental or surgical procedures. Poor oral hygiene and periodontal disease may be the precursor to bacteremia with viridians strep especially in immunocompromised patients.)

All of the following factors offer a protective effect, delaying symptoms in persons with hereditary hemochromatosis (HH) EXCEPT?

regular use of multivitamins with iron

What is the most frequent genotype among Rho (D) negative persons?

rr

How would you describe the appearance of a mature monocyte seen in a Wright's stained peripheral blood smear?

Which porphyria has a defect in the ferrochetalase enzyme within the RBC, causing redness, edema and burning when exposed to light?

The appearance of the bacterial colonies growing on the surface of the Hektoen Enteric (HE) agar plate shown in the photograph to the right are consistent with all of the following bacterial species EXCEPT:

shigella soneii

The body of an adult tapeworm is known as the:

strobila

All of the following carbohydrates are considered reducing sugars EXCEPT:

sucrose

Which of the following gene translocations is associated with Acute Promyelocytic Leukemia (APL)?

t(15;17)

Which chromosome abnormality is most commonly associated with CML?

t(9;22)

Which of the following terms would be used to express the middle value in a series of results?

the median

In which organ or tissue do T-lymphocytes mature?

thymus

A cerebral spinal fluid was collected from a lumbar puncture to evaluate for possible meningitis. Four tubes were collected and sent to the microbiology laboratory for analysis. Which of the four tubes is preferred for microbiology studies?

tube 2

What is the maximum recommended amount of time a specimen without additive or anticoagulant should be allowed to clot before centrifugation?

two hours

The overtime budget for the laboratory is $37,883, but $60,419 has already been spent. What percent over budget does this represent?

59%

Which of the following is NOT considered necessary for making a diagnosis of hereditary hemochromatosis (HH)?

A positive family history

The serum of which of the following individuals may agglutinate group A1 cells?

A2 individual

A Comprehensive Metabolic Panel (CMP) consists of which group of tests?

Albumin, Glucose, BUN, Creatinine, Sodium, Potassium, Chloride, CO2, AST, ALT, Alkaline Phosphatase, Total Protein, Total Bilirubin, Calcium

Illustrated in the image to the left are 3-day old colonies growing on blood agar. The colonies are gray and spreading, and a sweet, fruity odor is noted. Plump, Gram-negative coccobacilli are seen on Gram stain. This isolate belongs to the motile, non-fermenters with peritrichous flagella. Carbohydrate utilization is asaccharolytic. Nitrite reduction produces gas and citrate is positive. From the multiple choices, select the name of this isolate.

Alcaligenes faecalis

All of the following statements about using bar-coded labels on specimens are true, EXCEPT?

All possibility of errors is eliminated.

Alder-Reilly inclusions may be found in which cell type(s)?

All types of mature white blood cells

Which of the following refers to the most common procedure for donating whole blood for use by the general population?

Allogeneic donation

Septic arthritis can be caused by Neisseria meningitidis, Streptococcus pyogenes, and Streptobacillus moniliformis. Diagnosing these organisms from joint fluid can be difficult because they may not grow in culture. Which of the following contributes to the cause of non-culturable septic arthritis?

Antigen/Antibody complexes accumulate in the joint

Which of the following is used to determine if a person has had a recent infection with Strep pyogenes?

Antistreptolysin O (ASO)

What is this suspicious form seen in a stool sample that measures 10 µm in diameter?

Artifact

What are the white blood cell inclusions that are indicated by the arrows in this image?

Auer rods

Which of the following group B antigens is generally associated with a mixed field reaction?

B3

Which of the following instrument-based systems are considered a continuous monitoring system?

BACTEC

Which polyomavirus is commonly associated with nephritis post-transplant?

BK polyomavirus (BKV)

What biosafety level (BSL) is required by the Centers for Disease Control and Prevention (CDC) for working with Mycobacterium tuberculosis if there is a possibility that aerosols might be released?

BSL 3

Which of the following parasites has a life cycle resembling that of the Plasmodium species?

Babesia microti

Which of the following organisms would give the gram stain microscopic results seen in this image?

Bacillus anthracis

Which bacterial species is most likely represented by the spreading, gray-white, beta-hemolytic colonies with a "frosted glass" appearance as illustrated in the photograph?

Bacillus cereus

Atherosclerosis is due to which of the following?

Build-up of plaque

Which of the following laboratory tests is used as an indicator of inflammation?

C-Reactive Protein (CRP)

Which complement component is found in both the classical and alternative pathways?

C3

The media used to isolate Legionella pneumophilia should contain which of the following additives?

Charcoal and yeast extract

The buffering capacity of blood is maintained by a reversible exchange process between bicarbonate and:

Chloride

A patient presents at the ER with severe diarrhea and dehydration, marked with rice-water stool. The patient has a travel history to India. A stool culture was obtained and grew comma-shaped, gram-negative bacilli. What is the most likely diagnosis for this patient?

Cholera

All of the following tests are part of the Electrolyte Panel, EXCEPT?

Cholesterol

Identify the urine sediment elements indicated by the arrow in the illustration:

Cholesterol crystals

A lipid panel consists of which of the following tests?

Cholesterol, Lipoprotein, HDL, Triglycerides

A lipid panel is composed of which of the following tests?

Cholesterol, lipoprotein, high-density cholesterol (HDL), triglycerides

Where does most cerebrospinal fluid originate?

Choroid plexus

This member of the Enterobacteriaceae was isolated from a stool specimen and gave the following biochemical reactions: TSI: K/A, no gas, no H2S Citrate: Positive Lysine decarboxylase: Negative Ornithine decarboxylase: Positive Indole: Negative This organism agglutinated in Group D Shigella antiserum. All of the above biochemical results are consistent with this identification, EXCEPT?

Citrate

A D-positive mother with a D-negative fetus eliminates the possibility of HDFN due to the:

D antigen

Which of the following patients is eligible for RhIG?

D-negative woman, 28 weeks gestation, no unexpected antibodies

What is the most common Rh haplotype among whites?

DCe

What is the composition of Howell-Jolly bodies?

DNA

Zinc deficiency in the elderly is often caused by:

Decreased intake and absorption

Illustrated in the photomicrograph is a small 8 µm ameboid-like oval trophozoite with a pair of nuclei each with prominent karyosomes. This trophozoite was observed in a stool specimen of a person with intermittent persistent diarrhea, loss of appetite, and flatulence. What is the presumptive identification of this organism?

Dientamoeba fragilis

A two-year old male patient has the following hemogram results: Hgb = 6.7g/dLHct = 20% Platelets = 355 x 109/L RBC = 3.0 x1012/L WBC = 8.75 x 109/L Differential = 6% eos, 20% segs, 68% lymphs, 6% monos Which of the following is the MOST likely diagnosis?

Dietary iron deficiency

Stool is the specimen of choice for the recovery of which one of the following parasites?

Diphyllobothrium latum

Essential components of compatibility testing include all of the following EXCEPT:

Direct Antiglobulin Test (DAT)

A nasopharyngeal specimen is processed and fixed onto a microscope slide. Next, the fluorescein-conjugated antibody is added to the slide. The specimen is incubated with the labeled antibody, washed, and then observed for fluorescence. Which of the following techniques best describes this process?

Direct immunofluorescence

Which of the following is the most common reagent source for anti-A1?

Dolichos biflorous seed extracts (lectins)

What cytoplasmic inclusion is indicated by the arrow in this image?

Döhle body

The normal De Ritis ratio (AST/ALT) should be:

Equal or < 1

Which organisms are the appropriate stock cultures for quality control testing of oxidase production?

Escherichia coli / Pseudomonas aeruginosa

Identify the Light intensity control and Stage controls.

H, F

From the four specimens listed for Gram stain and culture, which one should be rejected?

Expectorated sputum in a sterile container for anaerobic culture

The properties of an antibody class are defined by the:

Fc end of the molecule

If a phlebotomist is vigorously shaking the blood tube to mix it, which of the following pre-analytical errors can occur?

Hemolysis

A specimen for potassium was drawn in a tube without anticoagulant at 6 AM during a traumatic patient stick. The results were 6.0 mmol/L. Why are the results elevated?

Hemolysis of the RBC's

Which situation will cause a positive DAT (direct antiglobulin test) in the newborn?

Hemolytic disease of the fetus and newborn

Xanthochromia in CSF is characteristic of:

Hemorrhage

Which of the formulas given below is used to calculate MCHC- mean cell (corpuscular) hemoglobin (Hgb) concentration?

Hgb (g/dL) x 100/ Hematocrit (%)

False-positive tests for protein on a urine reagent strip may be caused by the following conditions, EXCEPT:

High protein diet

A high-sensitivity C-reactive protein (hs-CRP) test result of 5 mg/L (normal = < 1 mg/L) may indicate which of the following?

High risk for future CVD

Which of the following is a biomarker in acute coronary syndrome?

High sensitivity-C-reactive protein (hsCRP)

The most common testing method for vitamin E determination is by:

High-performance liquid chromatography (HPLC)

Corynebacterium jeikeium is often associated with which of the following conditions?

Infected indwelling devices in AIDS patients

The adult liver fluke, Fasciola hepatica can be identified by its relatively small size and cone-shaped anterior extension (arrow). How do humans become infected with this organism?

Ingestion of poorly cooked water plants

The risk of laboratory-acquired infections with viruses, such as HIV-1, is a critical consideration. All of the following routes are considered modes of transmission with infected blood or other body fluids, EXCEPT?

Inhalation (airborne transmission)

Which of the following anthrax infections is also known as "woolsorter's disease"?

Inhalation anthrax

Penicillin G, ampicillin, carbenicillin, and methicillin are antimicrobial agents whose MAIN biological function is to:

Inhibit cell wall synthesis

Streptococcus pneumoniae can be isolated from peritoneal fluid and identified by molecular testing. One method of testing used to identify Streptococcus pneumoniae is molecular hybridization. All of the following steps must be followed to ensure that false-positive results do not occur when testing for Streptococcus pneumoniae EXCEPT?

Inhibitors must be removed

Which fungal culture media is used for the primary recovery of pathogenic fungi exclusive of dermatophytes?

Inhibitory mold agar

What is the mechanism of action for methotrexate?

Inhibits DNA synthesis

The granules of Basophil contain which of the following?

Initiators of allergic inflammation

Which branch of the immune system has an immediate response on first exposure to a foreign antigenic stimulus?

Innate

Skin and mucous membranes are important components of what part of the immune system?

Innate immunity

The vast MAJORITY of would-be invaders are killed or inactivated primarily by which part of the immune system?

Innate immunity

Which spirochete is a causative agent of human disease and is acquired through contact with the urine of carrier animals?

Leptospira interrogans sensu stricto

Methotrexate is a highly toxic medication that blocks DNA synthesis in all cells. What medication needs to be administered after methotrexate to prevent cytotoxic effects in normal cells of the body?

Leucovorin

Bone marrow examination may be used to aid in the diagnosis of which of the following?

Leukemia

Which of the following stains is used to differentiate a neutrophilic leukemoid reaction from chronic myelogenous leukemia (CML)?

Leukocyte alkaline phosphatase (LAP)

Which of the following is most likely to interfere with the measurement of hemoglobin?

Lipemia

A sputum specimen submitted for culture is brick red in color. The organism causing this sputum color also has a distinct polysaccharide capsule and is non-motile. What organism can be associated with this sputum?

Klebsiella

The reactions in a portion of the API strip, shown in the photograph, most closely identifies which organism?

Klebsiella pneumoniae

The top image shows a MacConkey agar plate on which mucoid, lactose fermenting colonies are present. The bottom image is the bacterial isolate streaked on the surface of a Mueller-Hinton agar plate. The disk to the left is ceftazidime, and the disk to the right is a combination of ceftazidime + clavulanic acid. Note the larger zone of inhibition around the combination disk on the right, an indicator for extended-spectrum beta-lactamase (ESBL) activity. Which of the following organisms fits this situation?

Klebsiella pneumoniae

Which of the following conditions would have macrocytic red cells?

Liver disease and Vitamin B12 deficiency

What is the purpose of the Total Complement Activity (CH50) assay?

Measures classic or terminal complement pathways

Urine test strip positive for blood may indicate all, EXCEPT:

Meat diet

Which of the following will cause an acidic urine pH?

Meat-filled diet

If test scores are significantly skewed, which measurement of central tendency is best?

Median

What is the platelet precursor cell found in the bone marrow?

Megakaryocyte

Which of the following sugars has been fermented when a gram-negative bacillus produces an alkaline slant and an acid butt on triple sugar iron agar (TSI)?

Glucose

This member of the Enterobacteriaceae gave these reactions: BiochemicalReaction H2S(TSI) Negative Ornithine Negative Lysine Positive Citrate Positive Urea Positive Phenylalanine Negative Motility Negative What is this organism?

Klebsiella pneumoniae

What is the probable identity of an organism isolated from a wound culture that demonstrates the following test results? Phenylalanine Deaminase = negative TSI = A/A gas Citrate = positive Lysine = positive SIM = negative/negative/negative Urea = weakly positive

Klebsiella pneumoniae

Which of the following tests is suitable for quantifying the size of fetomaternal hemorrhage (FMH)?

Kleihauer-Betke test

Which of the following can cause a false positive result for ketones?

L-Dopa metabolites

Which of the following tests on amniotic fluid would be included when assessing fetal lung maturity:

L/S Ratio

The anaerobe producing the double zone of hemolysis, seen in this photograph, can be presumptively identified as Clostridium perfringens. The enzyme producing the outer zone of hemolysis is:

Lecithinase

The beta hemolysis seen around the opaque, flat, spreading colonies illustrated in this photograph of a 5% blood agar plate is helpful in differentiating Bacillus cereus (hemolytic) from Bacillus anthracis (not hemolytic). Each of the following characteristics is also helpful in differentiating these two species EXCEPT:

Lecithinase production

The cells included in the composite image were found in the peripheral blood smear of a patient with the following results: total WBC of 21.5 x 109/L. Differential count: metamyelocytes 4 band neutrophils 16 segmented neutrophils 48 monocytes 6 eosinophils 1 basophils 1 lymphocytes 24 This hematologic picture is most consistent with:

Left shift

The Gram stain of a sputum specimen is unremarkable but the culture reveals many small gram-negative rods that are able to grow only on charcoal yeast extract agar. What is the MOST likely diagnosis?

Legionnaire's disease

This suspicious form was recovered from a bone marrow biopsy and measures 4 µm in length. What is the MOST likely identification of this organism?

Leishmania amastigote

Which one of the following adipocyte products is an important messenger in metabolism, signaling the hypothalamus that there are changes in fat stores?

Leptin

Hematocrit is:

Percentage of blood made up of red blood cells

Which components characterize the second line of human body defense against microbial pathogens?

Phagocytic neutrophils and macrophages

What has happened in an antibody titer, if tubes #5-7 show a stronger reaction than tubes #1-4?

Prozone reaction

When working up a culture, Scedosporium species may presumptively be identified if colonies have a "house mouse" gray appearance and microscopically observed are long delicate condiophores from the tips of which are produced a single oval conidium ("lollipop"). In some cases, further examination of the microscopic mount may reveal the branching, sheath-like arrangement (graphium form) of conidiophores as seen in the top image, and/or the spherical conidia-containing ascospores seen in the lower image. From the multiple choices, indicate the fungal species to be recognized.

Pseudoallescheria boydii

A gram-negative rod isolated from a burn patient produced a bluish-green pigment and the following test results: Characteristic fruity odorTriple sugar iron (TSI) agar: Alkaline/alkalineMotility: PositiveOxidase: PositiveOxidative/fermentation (OF) glucose: Oxidative utilization only What is the MOST probable genus of this isolate?

Pseudomonas

A flagellar stain would show peritrichous flagella for each of the following organisms except:

Pseudomonas aeruginosa

The detection of a distinct odor is often helpful in the presumptive identification of bacterial culture isolates. Which bacterial species produces a distinctive odor of grapes?

Pseudomonas aeruginosa

Which of the following tests require a 72-hour stool (fecal) collection?

Quantitative fecal fat

What characteristic is usually associated with IgM antibodies directed against red cells?

React best at room temperature

The lowest taxonomic level in the cognitive domain is:

Recall

The term TITER (as it applies to the measurement of antibodies) is best defined as:

Reciprocal of maximum reactive dilution

Which of the following is a key benefit that molecular methods can offer over culture methods?

Reduced turnaround time

All of the following statements are TRUE regarding storing and handling of urine reagent strips if you are using a manual method (visually read reagent strip), EXCEPT:

Remove the cap of the test strip bottle and leave it off until testing is completed.

The biochemical testing results of an alpha-hemolytic streptococcus show that the isolate is bile esculin-positive, but does not grow in the presence of 6.5% NaCl. The MOST PROBABLE identity of this isolate is:

Group D streptococcus, non-enterococcus

What blood group are the RBC screening cells that are used for antibody screens?

Group O

Which blood group is considered the "universal" donor when transfusing Red Blood Cells?

Group O

Which of the following contains all the possible phenotypes that could be the result of parents who are group O and group A?

Groups A or O only

All of the following descriptions about Pasteurella species are true, EXCEPT:

Grows well on MacConkey agar.

Sheep's blood agar is least suitable for the growth of which of the following organisms?

H. Influenzae

Six months ago, a 55-year-old female had a fasting plasma glucose (FPG) greater than 200 mg/dL on two occasions. She was diagnosed as having type 2 diabetes and treatment was started. Along with routine FPG's which of the following tests would most likely be used to monitor glycemic control?

HbA1C

What additional information is required on a label or tie tag of an autologous unit?

Identification of the recipient

Reviewing a Wright stained blood smear with the 10x objective is beneficial for all of the following EXCEPT?

Identifying abnormal or immature cells

According to the OSHA Bloodborne Pathogens Standard, when should work surfaces in the laboratory be decontaminated with an appropriate disinfectant?

If blood or other potentially infectious material has spilled on the work surface

The antibody mediator associated with a Type I Hypersensitivity Reaction is:

IgE

Which immunoglobulin is associated with anaphylactic shock?

IgE

Which of the following immunoglobulin classes is chiefly responsible for the degranulation of mast cells and basophils?

IgE

A secondary immune response is generally associated with which of the following antibodies?

IgG

Antibodies of which of the following immunoglobulin subclasses is predominantly associated with the secondary antibody response?

IgG

Respiratory acidosis is associated with:

Increased pCO2

How does hydroxyurea aid in the treatment of sickle cell disease?

Induces increased production of HbF

Delayed hemolytic transfusion reactions are usually caused by antibodies directed against what blood group system?

Kidd

Vitamin D is formed in the skin and hydroxylated in two places before it is in its active form of 1,25-dihydroxy vitamin D. Where does the second hydroxylation occur?

Kidneys

Carbolfuchsin binds to mycolic acid in cell walls with high lipid/wax content and is retained after decolorizing, preventing counterstaining with methylene blue. This principle describes:

Kinyoun acid-fast staining

A group of Canadian travelers was visiting the island of Kauai in Hawaii for a summer vacation. They noticed a cave in the middle of the island that contained a 10 ft deep pool of water. Much to their surprise, they were the only individuals enjoying this newly discovered swimming hole. A few days later, all swimmers became ill. Their physician believed they all had Weil disease. Which of the following organisms can cause this disease?

Leptospira interrogans

If a patient's WBC is 50,000/mm3, what test should be ordered to determine if this is a leukemoid reaction or a chronic myelocytic leukemia?

Leukocyte alkaline phosphatase stain

All of the following would be included in a hemolytic transfusion reaction investigation, EXCEPT:

Leukocyte antigen studies

Which of the following is characteristic of an exudate effusion?

Leukocyte count greater than 1,000 WBC/µL

Which of the following enzymes mediates the joining of primers during the ligase chain reaction (LCR) amplification technique?

Ligase

Nephelometry and turbidimetry measure different properties of:

Light transmission and scatter by particles in suspension

Which of the following blood collection tubes is appropriate for coagulation testing?

Light-blue top tube

What is the primary target of HBV?

Liver

A CSF specimen was sent to the laboratory for analysis. A glucose, protein, and cell count were performed. Based on the following results, what would be the probable cause? AnalyteResultGlucose50 mg/dLProtein25 mg/dLLeukocytes2 cells/mm3

Normal CSF

A patient with suspected intraabdominal malignancy had a paracentesis procedure performed. The fluid was then sent to the laboratory for examination and fluid differential. Which cells predominate in this cytospin field from the patient's sample?

Plasma cells

Parasites that may be found in urinary sediment include all of the following EXCEPT:

Plasmodium malariae

In which developmental stage do red blood cells begin forming hemoglobin in amounts large enough to be visualized on a Wright-stained bone marrow aspirate smear?

Polychromatic normoblast

This image represents a field of red blood cells (RBCs) stained using a supravital stain. How would these cells appear if viewed on a Wright-stained smear?

Polychromatophilic RBCs

While performing a peripheral blood smear review, you observe the cells indicated by the arrows on this Wright-Giemsa stained slide. How would you classify these cells?

Polychromatophilic RBCs

Which glycogen storage disorder is caused by mutations in the GAA gene?

Pompe disease

The scatterplot that is pictured represents results from a normal peripheral blood sample that was analyzed using flow cytometry. The cell that is indicated by the arrow on the right would fall into which of the circled areas?

Population C

The following parasites have been matched correctly with their common names EXCEPT:

Pork tapeworm =Taenia saginata

What is the group of diseases called where there is a deficiency in the enzyme(s) involved in hemoglobin synthesis?

Porphyrias

Which finding best distinguishes immune hemolytic anemia from other hemolytic anemias?

Positive DAT

To ensure precision and accuracy of the reagent strip tests used for chemical analysis of urine, it is necessary that:

Positive and negative controls are run on a daily basis and when a new bottle of test strips is opened

How should the production of a blue or purple color on the test pad of an Ictotest® be interpreted?

Positive for bilirubin

Identify the cell in this image:

Promyelocyte

Which of the following is most likely to interact with arterial walls, leading to deposition of cholesterol, and initiating or worsening atherosclerosis?

Small dense LDL

Why are small dense LDL molecules more atherogenic?

Small dense LDL molecules have greater mobility in the endothelium and arterial wall.

The cell diameter of a normal RBC is similar in size to the nucleus of a __________________.

Small lymphocyte

Which of the following cells is characterized by a thin rim of cytoplasm around the nucleus?

Small lymphocyte

The risk of exposure of a person to an infection of tuberculosis (TB) disease may be related to which of the following:

Socially disadvantaged

Which of the following parasites may be contracted by swimming in contaminated water?

Schistosoma japonicum

Baseline testing for latent Mycobacterium tuberculosis infection (LTBI) is recommended for all newly-hired healthcare workers. In which of these cases would a tuberculin skin test (TST) be repeated a second time, one to three weeks after the first test?

The first test is negative and the new employee has never been tested for LTBI before.

If a plasma glucose specimen is not centrifuged, how will the test results be affected?

The glucose test result may be decreased.

The qualitative differences between A1 and A2 phenotypes includes all of the following EXCEPT:

The lack of agglutination of patient red cells with anti-A reagent.

The deficiency in which fat-soluble vitamin is associated with night blindness?

Vitamin A

Which one of the following tests would be positive in the presence of Klebsiella spp.?

Voges-Proskauer test

The term pharmacokinetics BEST describes:

What the body does to the drug

Which color(s) are associated with biohazardous containers?

Which components characterize the second line of human body defense against microbial pathogens?

The three main types of peripheral blood cells are:

White blood cells, red blood cells, and platelets

In which of the following scenarios is instrument calibration verification not needed according to CLIA?

With each run performed on the instrument.

Which of the following errors occurs in the pre-analytical phase?

Wrong test ordered

The image shows a variety of colony elevations. Which colony elevation description matches with figure B in the image?

flat

In rapid carbohydrate fermentation testing, Neisseria gonorrhoeae ferments the following sugar(s):

glucose only

Which one of the following organisms is sensitive to bacitracin:

group a strep

Pseudomonas aeruginosa was isolated from a sputum specimen. Susceptibility testing was performed and the microbiologist determined that further evaluation was needed before reporting out the susceptibilities. Which of the antibiotic results would indicate a need for further evaluation?

Amikacin resistant, Gentamicin or tobramycin sensitive, Carbapenem-resistant

Which of the following is a true statement about aminoglycoside antibiotics?

Aminoglycosides can be toxic antibiotics used to treat bacterial infections when such bacteria are resistant to less-toxic antibiotics.

Identify the species associated with these parasitic forms that are typically found in a stool specimen.

Ancylostoma duodenale

Which one of the following methods is the best way to confirm your patient's identity?

Ask the patient to state his/her name, and check the patient's armband, comparing the full name and medical record number to that shown on your preprinted collection list.

This image is of a mold colony on an agar plate is most representative of which of the following molds?

Aspergillus niger

Which of the following is true concerning atherosclerosis?

Atherosclerosis is the deposition of plaques containing cholesterol and lipids on the innermost layer of the walls of large and medium-sized arteries.

All of the following methods can be used to transport carbon dioxide to the lungs EXCEPT:

Attached to neutrophils

What organism produced outside a bioterrorism event causes glanders disease?

Burkholderia mallei

Which bacterial species is most likely represented by the spreading, wrinkled, non-hemolytic colony appearance as illustrated in the photograph?

Burkholderia pseudomallei

Which of the following conditions is associated with the Epstein-Barr Virus?

Burkitt's lymphoma

Which of the following images shows a Gram stain that is consistent with Nocardia species?

C

Which of the following is the "activation unit" in the classical complement pathway?

C4, C2, C3

This species of Candida accounts for around 20 percent of urinary yeast isolates.

Candida glabrata

Each of the tests below is appropriately matched with a chemical responsible for producing a positive result, EXCEPT:

Citrate - alpha naphthol

Which of the following describes the quality standards that all laboratories must meet to ensure testing accuracy, reliability, and timeliness no matter where a patient sample is tested?

Clinical Laboratory Improvement Amendments

A 65-year-old Asian female presented to the emergency room complaining of severe abdominal pain, fever, and diarrhea. Examination revealed an enlarged liver that was tender to the touch. The patient noted that she had worked in a fish processing plant for several years before moving to the United States. A significant component of her diet consisted of raw fish. Both stool and duodenal contents were collected and sent to the laboratory for culture and parasite examination. The bacterial cultures were unremarkable. This suspicious form, measuring 27 µm by 14 µm, was found microscopically in both specimens. What organism is the most likely cause for this patient's symptoms?

Clonorchiasis

A sample from a wound infection, suspected case of gas gangrene, was cultured and showed anaerobic colonies that are small, smooth, light gray-yellow, and surrounded by a distinctive double zone of beta hemolysis (arrows). Gram stain, as illustrated in the lower photomicrograph, are large, non-spore forming, "box-car" shaped gram-positive bacilli. What is this organism that is also related to outbreaks of food-borne diarrhea following ingestion of contaminated meat products?

Clostridium perfringens

An India Ink preparation is used to identify:

Cryptococcus neoformans in CSF

The curved, multi-celled, dark brown staining macroconidia divided by transverse septa as seen in this photomicrograph is characteristic of which of the following species of dematiaceous fungi?

Curvularia species

All of the following statements concerning biochemical reactions are true, EXCEPT:

Escherichia coli is positive for utilization of sodium citrate, as depicted in tube A.

What is the role of the Joint Commission (JC)?

Establish standards for the operation of healthcare facilities and services.

Recently the American Diabetes Association (ADA) recommended reporting which of these values in order to correlate with hemoglobin A1C (HbA1C) as a further indicator of glycemic control?

Estimated average glucose

The Entero-Test may be helpful to identify which of the following parasites from duodenal contents?

Giardia duodenalis

This photomicrograph is of an oval 10 µm cyst found in a third stool specimen from a patient with intermittent diarrhea. Note the thin, smooth outer membrane. Two nuclei are observed within, with large, eccentrically placed karyosomes devoid of a chromatin ring. Four curved fibrils are also present. From the observation of this cyst, select the presumptive identification.

Giardia duodenalis

Which of the following tests are necessary to confirm the condition suggested by the morphology shown in this image?

Hemoglobin electrophoresis for the presence of Hb S

Which of the following is an FDA-approved therapeutic agent for treating sickle cell patients?

Hydroxyurea

A patient is admitted to the emergency room with severe lethargy, glossitis, and muscle dysfunction. After the physician orders a complete blood count with differential, the hematology technologist observes cells matching the image to the right. Which condition is most consistent with the clinical and laboratory findings?

Iron deficiency anemia

The abnormal RBCs seen in this illustration are indicative of:

Iron deficiency anemia

Which of the following criteria for donor RBC should be used for an exchange transfusion related to both HDFN due to anti-D and ABO HDFN?

Irradiated

A characteristic of a good cardiac biomarker is that:

It can be detected even if it is present in a LOW concentration in the peripheral blood.

A laboratory developing a new testing procedure runs the test on 1,000 patients. Later confirmation shows that the experimental procedure produced remarkably few false-negative results but many false-positive results. Which of the following statements is true regarding the experimental procedure?

It has high sensitivity

Which of the following statements is true regarding macrophages (histiocytes)?

It is a type of monocyte that has entered the tissues via diapedesis.

Why might serum ferritin (SF) alone be considered a less than optimal screening test for hereditary hemochromatosis (HH)?

It is an acute phase reactant that is frequently elevated in a variety of clinical conditions.

All of the following statements concerning Clostridioides difficile are true, EXCEPT?

It is an aerobic organism.

Which of the following statements about the test for C-reactive protein (CRP) is TRUE?

It is an indicator of an inflammatory condition

Which one of the following statements concerning Tamm-Horsfall protein is FALSE?

It is the protein that is predominantly detected by a urine reagent strip test.

What information can be added as a unique patient identifier to properly complete the labeling in the image?

Jane Doe, Hospital ID # 7890123

Which of the following characteristics is consistent with the Kidd blood group system?

Jk(a-b-) red cells are resistant to lysis in 2 M urea

All of the statements below about synovial fluid are true EXCEPT:

Joint fluid resembles plasma in viscosity

What reagent should be used in conjunction with calcofluor white when preparing the fungal slide for interpretation?

KOH

The immunoglobulin molecule is made up of both heavy and light chains - the light chains can be comprised of which of the following?

Kappa or lambda

A patient has a WBC count of 4.0 x 109/L. Giant platelets, as seen in the image, are observed on the Wright stained peripheral smear. Blue-staining inclusions are also observed in several of the neutrophils. Considering the morphology noted by the white arrow, what is the identity of this patient's blue-staining WBC inclusions?

May-Hegglin anomaly inclusion

Louisa is a healthy 24-year-old woman that wants to donate blood today. Her hemoglobin, temperature, blood pressure, and pulse are all within the acceptable limits for donating. She's in good health and not taking any medications. She recently divorced her spouse who was a hemophiliac that regularly received factor concentrates. Can Louisa donate blood today?

Maybe, depending on Louisa's last sexual contact with her ex-spouse.

The following image is from a cytocentrifuged preparation of serous fluid. The arrow is pointing to which of the following:

Mesothelial cell

Each of the cells below can be found in all types of body fluids EXCEPT:

Mesothelial cells

On a Cytospin preparation from a pleural fluid specimen, 50% of the cells have the following characteristics: - uniform, regular arrangement- some cells resemble a "fried egg"- multiple nuclei- smooth nuclear outline and homogeneous chromatin- when present in clumps, there are clear spaces between the cells ("windows") How should these cells be classified?

Mesothelial cells

Which of the following cells is the most common nucleated cell in normal adult bone marrow

Metamyelocyte

Which assay, using 24-hour urine, is considered the BEST single screening test for pheochromocytoma?

Metanephrines

What is the first step of the ethyl acetate concentration procedure?

Mix fresh stool specimen with 10% formalin and strain to obtain a sediment

The numeric value that occurs most frequently in a sample distribution is called the:

Mode

All of the following are granulocytes EXCEPT:

Mott cell

Illustrated in the top photograph is a 3 day old colony incubated at 30o C, with a border to border white hair-like mycelium with peripheral borders of brown and brown-black pigmentation. Although consistent with one of the Zygomycetes, a genus/species identification cannot be made. Observed in the high power photomicrograph is a lactophenol blue stained mount of a fruiting head in which a non-septate hypha terminates in a bulbous columella (arrow) that extends into a spherical sporangiophore. Rhizoids were not observed. From these observations, select from the multiple choices the name of this isolate.

Mucor species

Critical results from the microbiology department must be communicated to the clinician immediately. All of the following results are considered critical except

Negative acid-fast stain

Which statement best describes the relationship between dietary iron and iron absorption in a healthy individual?

Only a small percentage of dietary iron is absorbed and used.

Microorganisms that may be best visualized with a Trichome stain (shown here) include:

Protozoan parasites

When scanning a Wright's stained blood smear, which of the following would describe the optimum area to begin your blood smear analysis and differential white blood cell count?

RBCs are lying singly, barely touching, with occasional overlapping.

What is the distance the solute migrates divided by the distance the solvent migrates in thin-layer chromatography (TLC)?

Rf

All of the following are indications for administering Rh immune globulin (RhIg) to an Rh negative woman, EXCEPT?

Rh negative fetus

To prevent Rh alloimmunization in pregnancy between mother and fetus, it is recommended to initiate RhIG prophylaxis when?

Rh-negative pregnant females, antenatal in the third trimester.

In the hookworm life cycle, which of the following is considered the noninfective larval form?

Rhabditiform

Illustrated in the high-power photomicrograph is a large 140 µm ovum from a stool specimen preparation. The adults reside in the portal veins of the large intestine where ova are discharged into the lumen and the feces. Low-grade diarrhea may be present. Select the species of fluke related to this ovum.

Schistosoma mansoni

This parasitic egg that was found in a stool specimen measured 170 µm by 63 µm. Identify the parasite.

Schistosoma mansoni

Identify the cell indicated by the arrow.

Segmented neutrophil

If a TSI agar slant shows a neutral or slightly alkaline slant, acid butt, no/gas production, and no H2S production, the organism might be:

Shigella dysenteriae

Normal fecal specimens contain all of the following, EXCEPT:

Shigella spp.

Which of the following is an example of proper transfusion practices regarding fresh frozen plasma (FFP)?

Should be transfused within 24 hours of thawing

What is the characteristic RBC that is uniquely associated with HbSS?

Sickle cell (drepanocyte)

An 8-year-old girl's peripheral blood smear is shown in the image on the right. Based on the peripheral blood image, what is a likely diagnosis?

Sickle cell disease

Which of the following would best describe what you might observe after a traumatic CSF tap?

Significantly more RBCs were found in tube #1 vs. tube #3

Which latent transcription regulators are phosphorylated and activated by receptor serine/threonine kinases and carry the signal from the cell surface to the nucleus?

Smad family (Smad family is named after the first two proteins identified, Sma in C. elegans and Mad in Drosophila. Latent transcription regulators are phosphorylated and activated by receptor serine/threonine kinases and translocates into the nucleus.)

If septic arthritis is suspected, specimens from which of the following normally sterile sites should be collected to detect the microorganism causing the condition?

Synovial fluid

T lymphocytes are characterized by all of the following functions EXCEPT which?

Synthesize antibody

A combination of nontreponemal testing (VDRL or RPR) and treponemal testing (TP-PA or EIA) is used to confirm:

Syphilis

Which T lymphocyte expresses the CD8+ marker and acts specifically to kill tumor or virally-infected cells?

T cytotoxic

Contact dermatitis is mediated by:

T lymphocytes

What is the primary target of HIV (human immunodeficiency virus)?

T-helper cells

Which of the following tests is used to confirm a syphilis infection after another method tests positive for syphilis?

T. pallidum particle agglutination (TP-PA) test

Which antinuclear antibody (ANA) pattern is seen in the image on the right, which represents the result of an ANA test viewed using fluorescent microscopy? Note: (a) points to the nuclei of interphase cells, the primary consideration for discerning the ANA pattern and (b) indicates a metaphase mitotic cell.

There is no discernable pattern

When operating a centrifuge, what happens if the centrifugation time is reduced and the speed increased?

There will be incomplete separation of red blood cells and serum in the tube.

Organisms such as coagulase-negative Staphylococcus, Corynebacterium, and alpha streptococcus species are all found on the skin and represent normal skin flora. All of the following are functions of normal skin flora EXCEPT?

They aid in the transmission of nosocomial infections

All of the following are true about all members of the Enterobacteriaceae, EXCEPT:

They are all oxidase-positive

Histone proteins have the following characteristics and functions except:

They are complexes of DNA found in the nucleus of a eukaryotic cell.

What condition/situation is associated with the peripheral blood picture in the image on the right?

Transfusion dimorphism

All of the following are ways hemoglobin functions EXCEPT:

Transport Haptoglobin

What is the MOST probable condition when a clean-catch urine specimen gave these results: Appearance = cloudy, amber pH 5.0 Specific Gravity = 1.015, Protein = 1+ Glucose = negative Occult Blood = trace Nitrite = positive Microscopic = many bacteria and 30 - 40 WBC/HPF.

Urinary tract infection

Acetest® can be used to test for ketones in:

Urine, serum, and whole blood

Which of the following urine chemical reagent strip tests is based on the Ehrlich-aldehyde reaction?

Urobilinogen

A pleural fluid specimen is received in the laboratory for aerobic culture. The specimen measures about 0.5 mL. How should the tech process this specimen for culture?

Use a sterile pipette and add a drop of fluid per agar plate

All of the following viral causative agent and associated disease state combinations are correct EXCEPT:

Variola Virus: Polio

The end product of fermentative metabolism of the Escherichia, Salmonella, and Shigella genera is which of the following?

Various mixed acids

What causes polyacrylamide gels to have greater resolution of proteins than agarose gels?

Varying pore size in each polyacrylamide layer

Complaints of pain from a patient with sickle cell disease are most likely associated with which of the following conditions/events?

Vaso-occlusive crisis

Fire requires what three elements?

heat, fuel, oxygen

What is the name of the colorless end product of bilirubin catabolism?

Urobilinogen

Which nucleotide is specific to RNA?

Uracil

Which of the following drugs is used to treat congestive heart failure?

Digoxin

Identify the nucleated blood cell:

Monocyte

Which of the following is not a secondary structure of a protein?

Trimers

Which of the following genotypes results in a-thalassemia with HbH disease?

-a/--

The volume of urine recommended for centrifugation for a microscopic examination is:

10 - 15 mL

Which of the following kappa / lambda ratios are found in normal B cells?

2 : 1

A computer monitor should be set approximately how many inches away from the eyes?

20 - 40 inches

Red blood cell units that contain CPD (citrate-phosphate-dextrose) can be stored for up to how many days?

21 days

Following a major trauma event, a family of four walks into your donor center hoping to donate blood. Which one of the following individuals may donate blood today?

A 22-year-old woman using oral contraceptives.

Which of the following patients is most at risk for hyperosmolar nonketotic coma?

A 70-year-old type 2 diabetic patient

Which of the following statements best describes a normal mature erythrocyte?

A biconcave disc, 6-8 µm in diameter, that stains pinkish-orange (salmon) in color

Which is the MOST accurate definition for an orthochromic normoblast or nucleated red blood cell (NRBC)?

A cell with a low nuclear-to-cytoplasmic ratio with pink cytoplasm and a heavily condensed chromatin pattern.

Your screen cells are 3+ at immediate spin and weak (W)+ at AHG. Your autocontrol is negative for both phases. Some of your antibody panel cells are 3+ at immediate spin and negative at AHG. What should you suspect?

A cold antibody may be present

What is the most prudent step to follow to select units for crossmatch after recipient antibodies have been identified?

Antigen type patient cells and any donor cells to be crossmatched

Which one of these physical exam results would cause a donor to be deferred?

A diastolic blood pressure of 110 mm Hg

Which one of the following statements best describes hemoglobin A1c (HbA1c)?

A glycated protein that is an index of carbohydrate management over 2 - 3 months.

In therapeutic drug monitoring, a person who is classified as an ultrarapid metabolizer (UM) would need __________ of a drug metabolized by that enzyme.

A higher dose

A hapten can BEST be described as:

A nonimmunogenic material capable of stimulating an immune response only when bound to a carrier protein

The following describes a clot-based test to assess Protein C function, as well as the results of someone's test which has a protein C deficiency.

A parital thromboplastin time (PTT) is performed using C-depleted normal plasma, a venom activator of protein C (such as Protac), and a heparin neutralizer. The clotting time would not be prolonged in someone with a protein C deficiency.

Which of the following is a possible type for an offspring from the mating of an O and an AB (non-cis) individual?

AO

Which two of the following tests are helpful for documenting previous streptococcal throat and skin infections?

ASO titer & anti-DNase B

Which of the following tests would be employed in order to detect neural tube defects?

Alpha fetoprotein

In which disorder may a mild anemia be present even with an increased red blood cell (RBC) count and normal adult hemoglobin electrophoresis?

Alpha thalassemia minor

Which alpha thalassemia shows no anemia or evidence of disease, although a slight decrease in the MCV and MCHC may be seen?

Alpha thalassemia silent carrier

This suspicious form, that measures 25 µm, was recovered in an eye fluid sample. It is associated with which of the following diseases?

Amebic keratitis

A 3-year old girl was brought to the ER with a temperature of 103ºF, lethargy, and cervical rigidity. Three tubes of cloudy CSF were delivered to the Lab, and preliminary test results showed: WBC: 4,500/µL Differential: 88% neutrophils Glucose: 15 mg/dL 12% lymphocytes Protein: 140 mg/dL Gram stain: No organisms observed From these results, what preliminary diagnosis SHOULD the physician consider?

Bacterial meningitis

Aspiration material recovered from a ruptured appendix abscess was inoculated to anaerobic culture media per laboratory protocol. Gray colonies (> 1 mm) grew anaerobically in 24 hours on BBE agar, blackening the media. The photograph illustrates the gram stain features of the organism. The bacterial species most likely associated with the abscess is:

Bacteroides fragilis group

All of the following are therapeutic drugs used to treat cardiac disease, EXCEPT?

Carbamazepine

A 25-year-old motorcyclist incurred superficial and deep penetrating lacerations of his right shoulder when thrown off his speeding vehicle into the ditch. The shoulder became painful and continued to swell over the next 24 hours. The bacterial species shown in the upper image was recovered after 36 hours under anaerobic incubation and the lower image illustrates the microscopic features. What is the most likely identification?

Clostridium perfringens

Each of the bacterial species listed below are correctly matched with its identifying characteristic, EXCEPT:

Corynebacterium urealyticum - urease negative

Storage lesion, or the biochemical changes in red blood cells during storage, will result in:

Decreased plasma pH

Which of the following is a drawback of point-of-care testing (POCT)?

Decreased precision in data

What is the correct order of steps that should be taken when a Six Sigma approach is used for problem-solving?

Define, Measure, Analyze, Improve, Control

Which action is a source of laboratory-related error?

Delayed critical result reporting

What is the anticoagulant found in the pink top tube?

EDTA

Which one of the following organisms is considered to be pathogenic?

Entamoeba histolytica

Which of the following forms of calcium is biologically active:

Free ionized calcium

All of the following red blood cell phenotypes will react with Anti-Fy3, EXCEPT?

Fy(a-b-)

Which of the following genotypes are known to be resistant to P. vivax (malaria) merozoites?

Fy(a-b-)

In the late 1990s, an outbreak of pneumonia was experienced by several high school students involved in a clean-up day held in a Midwestern community. One student experienced a severe disseminated infection. The colony growing on Sabouraud Dextrose with Brain Heart Infusion (SABHI) agar and incubated at 30o C and shown on the top photograph was recovered from an induced sputum specimen. The colony matured in 8 days, presenting a hair-like, cottony surface mycelium. The identification was confirmed by observation of the lactophenol tease mount obtained from the surface of the mycelium. From the list of choices below, select the name of this isolate.

Histoplasma capsulatum

What Anti-Nuclear Antibody (ANA) staining pattern is presented in the microscopic image?

Homogeneous

Review each matched pair. Each correctly matches the bacterial species with its Gram staining reaction and morphology EXCEPT:

Image B = Moraxella catarrhalis

Refer to the images on the right. Image A is delta-aminolevulinic acid viewed with ultraviolet light Image B is buffered charcoal yeast extract agar viewed with ultraviolet light Image C is MacConkey agar Image D is DNAse agar. Review each of the following names of the bacterial species with the appearance and reactions of the colonies seen on the media listed. Each is a correct match of the species with its appearance on the media EXCEPT:

Image C = Klebsiella pneumoniae

Which of the following statements best describes Rh antibodies?

Immune, IgG

All of the following conditions are associated with a polyclonal (broadbased) increase in gamma globulins except?

Immunodeficiency

For transfusion services in the United States, which of the following incidents must be reported to the Food and Drug Administration (FDA) because of a biological product deviation?

Incident C: The wrong specimen was used to crossmatch a unit and the unit was issued.

Which occurrence is a medical error?

Incompatible blood is transfused into a patient during surgery

The substance produced by the pancreas and released in the bloodstream to help glucose absorption by the tissues is called?

Insulin

Which of the following signal protein family acts via RTKs (receptor tyrosine kinases) and stimulates carbohydrate utilization and protein synthesis?

Insulin

Which of the following may be part of the master mix?

Internal Control

Which of these conditions could result in a microcytic anemia?

Iron deficiency

The condition which is highly associated with the oval-macrocytes and hypersegmented neutrophils found in this image is?

Megaloblastic Anemia

Which organism will infect humans and a wide range of host animals?

Mycobacterium bovis

Which of the following may be found in the cerebral spinal fluid (CSF) and measure approximately 15 µm?

Naegleria fowleri trophozoite

This smear shows:

No abnormality

The small club-shaped, "drumstick" nuclear appendage attached to one lobe of a neutrophil (marked by the blue arrow in the image) may be found in:

Normal female

This egg measuring 112 µm by 55 µm was recovered from a sputum sample. What is the identification of this organism?

Paragonimus westermani egg

HIV is known as a retrovirus because:

RNA is used as the template for DNA synthesis

Which adsorption technique removes cold (IgM) antibodies, particularly anti-I specificities?

Rabbit erythrocyte stroma (RESt)

Which type of error is associated with the precision of the method?

Random error

The 95% confidence limit refers to values that fall within:

Range equal to the mean plus or minus two standard deviations

Secondary hypercoagulable states can be caused by all of the following conditions EXCEPT:

Rapid weight loss

An urticarial transfusion reaction is characterized by which of the following?

Rash and hives

All of the following could explain why screening cell #2 has a stronger reaction than screening cell #1 EXCEPT:

Screening cell #1 has homozygous antigen for the reacting antibody whereas screening cell #2 is heterozygous.

Which one of the following statements is FALSE?

Secretory cells of the innate immune system are antigen specific.

Which of the following is true of the nuclear appendage seen in this image?

Seen only in women

The dimorphic fungi that may produce black, yeast-like colonies after prolonged incubation at 37°C is:

Sporothrix schenckii

The cell indicated by the arrow is a:

Squamous epithelial cell

Targeting the conserved regions of the 16S rRNA is commonly used to detect all, but which of the following organisms?

Streptococcus pneumoniae

Which of the following immunosuppressive drugs below has been associated with thrombus formation in patients with toxic levels?

Tacrolimus

What does the anatomical term "Distal" refer to?

The area farthest from the center of the body

Molecular methods offer improvements in sensitivity, but they are dependent upon what factor?

The availability of specific instrumentation

Who should a technologist contact if an organism is isolated that cannot be ruled out as a potential agent of bioterrorism?

The Laboratory Response Network (LRN) reference laboratory

What is edema?

The accumulation of interstitial fluid under the skin causing swelling.

What is the first step in the blood collection process?

The creation of a test request/requisition.

What is successful molecular identification of methicillin-resistant Staphylococcus aureus (MRSA) based upon?

The detection of the S. aureus genes (orfX, nuc, or coa) gene and the mecA gene.

All of the following statements are true regarding urinalysis reagent strips EXCEPT?

The expiration date on the bottle does not matter, as long as the controls are acceptable.

After irradiating a unit of packed red blood cells, how is the expiration date affected?

The expiration date shortens to 28 days from the date of irradiation or the original expiration date, whichever is first.

Which one of the following answer choices is correct when collecting capillary blood specimens for routine laboratory tests?

The first drop of blood at the puncture site should be wiped away.

Which of the following best describes a segmented neutrophil?

The nucleus contains 2-5 lobes connected by a thin filament.

The pathogenicity of Staphylococcus aureus, as well as the frequency with which this organism produces infections, can be attributed to all of the following EXCEPT:

The pathogenicity of Staphylococcus aureus, as well as the frequency with which this organism produces infections, can be attributed to all of the following EXCEPT: A porous cell wall

A patient has optimal LDL and HDL cholesterol values but a hsCRP test shows a value of 12 mg/L (low cardiovascular risk < 1.0 mg/L). Which of the following situations is most likely?

The patient has an underlying acute inflammation process.

When giving a whole blood donation to a patient, what consideration must be made in regards to the ABO/Rh compatibility between the recipient and the donor?

The recipient and donor ABO and Rh types must be identical.

Two techs performed E.coli O157:H7 serotyping on a suspected colony. The serotyping uses the O157 somatic antigen as the target for agglutination. One tech performed quality control and patient samples by using a suspension of the colony and latex reagent. The results reported by the tech were positive. The second tech also did the serotyping using quality control and patient samples by using a suspension of the colony, latex reagent, and isolate suspension with the negative control. The second tech's results were negative. Quality control passed in all testing. Why did the techs obtain different results for the serotyping?

The second tech tested the suspension with negative control

At an outpatient clinic, a patient comes in with an order for stool ova and parasites (O & P). How should this specimen be collected and transported for the best test results?

The specimen should be immediately placed in recipients containing polyvinyl alcohol (PVA) or formalin and transported to the laboratory.

According to the Michaelis-Menton kinetics theory, when a reaction is performed in zero-order kinetics:

The substrate concentration is in excess and the reaction rate is dependent on the enzyme concentration

A laboratory test flags a critical result. The technologist repeats the test, and it again flags as a critical result. Which of these scenarios correctly describes all the actions that need to be taken after this occurs?

The technologist phones the patient's caregiver, reports the result, and asks the caregiver to read back the information; documents his placement of the call, who was notified, time, and date of notification.

Serum TSH levels five times the upper limit of normal in the presence of a low T4 and low T3 uptake could mean which of the following?

The thyroid has been established as the cause of hypothyroidism.

Which of the following may cause a false-positive protein reading on a urinalysis dipstick?

The urine specimen is highly pigmented.

All the following statements about the urine specimen are true EXCEPT:

The urine specimen should always be centrifuged before performing the chemical reagent strip testing.

Which of the following drugs is used to treat asthma and COPD in patients with nocturnal symptoms or problems with inhalers?

Theophylline

A physician calls the lab to order a Hepatitis A IgG test but cannot find the code. What do you tell the physician?

There are no IgG-only tests for Hepatitis A, only a total with/without reflex to IgM

A physician discusses weight with an overweight 60-year-old female at her yearly physical appointment. The female exercises regularly and eats healthy most of the time. The physician suggests she decrease carbohydrate intake and decrease portion size at meals. Review patient vital signs and laboratory assay results to decide if a diagnosis of metabolic syndrome is appropriate using the NCEP: ATP III Diagnostic Criteria shown on the right. Height: 5' 7'Weight: 192 lbsBMI: 30.1Waist Circumference: 37 inchesBlood Pressure: 108/70 Fasting Blood Glucose: 92 mg/dLTotal Cholesterol: 172 mg/dLLDL-C: 112 mg/dLHDL-C: 46 mg/dLTriglycerides: 70 mg/dLhs-CRP: <1.0 mg/L Which of these statements regarding this patient is true?

This patient does not meet the criteria for diagnosis of metabolic syndrome.

A urine concentration test was performed on a patient after a fluid intake deprivation period with the following findings: After 12 hours Urine Osmolality = 850 mOsm/Kg After 16 hours Urine Osmolality = 850 mOsm/Kg

This represents normal kidney function.

What is the maximum concentration of a chemical allowable for repeated exposure without producing adverse health effects called?

Threshold limit value

Which of the following autoantibodies are found in a patient with Graves disease but not typically seen in Hashimotos thyroiditis?

Thyroid-stimulating hormone receptor antibodies (TRAbs)

The Lean approach seeks to streamline the process. What is the major measurement for Lean?

Time

Fluoroquinolones, such as ciprofloxacin, work by targeting:

Topoisomerases, inhibiting bacterial DNA replication

Mrs. Jones, a diabetic, is admitted to the hospital on November 25th. It appears that she has had an acute myocardial infarction (AMI), but did not feel the severe chest pain because of diabetic neuropathy. She has not felt well since November 20th and her physician believes she possibly had an AMI on November 20th.Assuming that the infarct was uncomplicated, which of the following cardiac biomarkers would most likely still be elevated and would assist in an AMI diagnosis?

Troponin T

Which troponin protein binds the troponin complex to tropomyosin?

Troponin T (TnT)

How would you describe the arrangement of RBCs on this slide?

True rouleaux

A 25 year old missionary who had recently returned from a trip to West Africa presented to the ED with fever and lymphadenopathy. A blood specimen was collected in EDTA and sent to the laboratory for possible parasitic infection. This image shows the parasite that was seen on the blood smear. What is the identification of this parasite?

Trypanosoma brucei

The stage of this organism has an undulating membrane that extends the length of the organism, a kinetoplast, basal body, nucleus, and flagellum. What is the most likely identification and stage of this organism?

Trypanosoma trypomastigote

The red ring at the top of the media in this tube indicates a positive indole reaction. The precursor substance in the medium from which the indole is produced is:

Tryptophan

All of the following are associated with immediate hypersensitivity, EXCEPT?

Tuberculin reaction

Which of these methods is used to identify prior exposure to Mycobacterium tuberculosis?

Tuberculin skin test (TST)

Which of the following best describes the Westgard multirule 22S?

Two consecutive data points fall outside +2 SD or -2 SD limits.

After incubation of whole blood or bone marrow in hypotonic buffer, centrifugation will differently separate cells out in which order?

WBCs will pellet to the bottom, and RBCs will remain suspended. (Red blood cells (RBCs) lyse before white blood cells (WBCs) in whole blood, making the WBCs heavier than the RBC membranes. When centrifuged, the heavier WBCs will pellet at the bottom of the tube, leaving the lighter RBCs in suspension. Special collection tubes with separators are needed to separate both RBCs and WBCs from plasma during centrifugation.)

Immunoglobulin M (IgM) is the characteristically overproduced gene product found in:

Waldenström's disease

Which of the following corresponds to a way to prevent infection with Sporothrix schenckii?

Wear gloves when gardening

From the choices listed below, select the characteristic that would immediately rule out a presumptive identification of Mycobacterium tuberculosis:

Yellow pigmented colonies after exposure to light

A small, microaerophilic, curved, gram-negative rod which was isolated from a diarrhea stool specimen gave the following results: Grew best at 42 ºC Oxidase positive Resistant to cephalosporin Susceptible to nalidixic acid Hippurate hydrolysis positive What is the MOST probable identification of this organism?

***Campylobacter jejuni*** Campylobacter coli Salmonella species Vibrio cholera

Which of the following is the most common etiologic agent identified in prosthetic valve endocarditis?

***Staphylococcus epidermidis*** Staphylococcus aureus Streptococcus sanguis Streptococcus mutans

Assuming 100% efficiency, what should the slope of a standard curve in a qPCR be?

-3.33 (The higher the Ct, the lower the concentration is, thus the slope will always be negative. Because PCR is exponential, after one cycle, there is twice as much amplicon. After two cycles, there is four times as much, and after 3.33 cycles there is ten times as much. Standard curves are made with ten-fold dilutions, so the slope between those will be -3.33.)

How many white blood cells (WBCs) would be considered normal for adult cerebrospinal fluid?

0-5 WBCs/µL

Determine the molarity of a solution of KOH that is contained in a Class A 1-L volumetric flask filled to the calibration mark. The content label reads 18 g KOH. The gmw of KOH=56.1 g/mol.

0.32 mol/L ((18 g KOH/ L) X (1 mol/ 56.1 g HCl) = 0.32 mol/L)

Which of the following is considered the major transport protein for vitamin A?

Retinol-binding protein

Acute immune thrombocytopenic purpura (ITP) is MOST OFTEN associated with which of these populations?

Children 2 - 6 years of age

In blood, bicarbonate leaves the red blood cell and enters the plasma through an exchange mechanism with:

Chloride

What is the normality (N) of a 500 mL solution that contains 8 g of H2SO4 (gmw=98 g/mol, valence=2)?

0.326 N ((8 g H2SO4/500mL) x (1 Eq/49 g H2SO4) x (1,000 mL/1 L))

What is the term used to describe inadequate blood supply that decreases availability of oxygen to the myocardium?

Myocardial ischemia

When identifying Mycobacteria by molecular methods, the most reliable gene sequence is:

16S

Serum proteins can be separated by cellulose acetate electrophoresis into how many basic fractions:

5 fractions

A 3-year old girl was brought to the ER with a temperature of 103ºF, lethargy, and cervical rigidity. Three tubes of cloudy CSF were delivered to the Lab, and preliminary test results showed: WBC: 4,500/µL Differential: 88% neutrophilsGlucose: 15 mg/dL 12% lymphocytesProtein: 140 mg/dLGram stain: No organisms observed From these results, what preliminary diagnosis SHOULD the physician consider?

Bacterial meningitis

The growth of yellow-white yeast colonies with a smooth or hair-like surface within 2 - 3 days on non-selective culture media generally requires further procedures to make a presumptive identification. Microscopic observation of the subculture of colonies growing on cornmeal agar is one such approach, as illustrated in the photomicrograph. Note the spider-like colonies that appear along the streak line. From the multiple choices, select the name of the presumptive yeast identification.

Candida parapsilosis

The morphology of yeasts inoculated to cornmeal agar can be helpful in making a species identification. The field of view shown in this photomicrograph is most consistent with the identification of:

Candida parapsilosis

A 64-year old man lost 15 pounds, became weaker, and his face became fuller with a ruddy complexion. His laboratory tests revealed: Serum cortisol 8 AM: 880 nmol/L, 138-635 nmol/L (reference range) 4 PM: 828 nmol/L, 83-414 nmol/L (reference range) After these results, an overnight dexamethasone suppression test gave a cortisol level of 773 nmol/L. The MOST probable diagnosis for this patient is:

Cushing's Syndrome

What substance is measured in a spectrophotometer when Drabkin's solution is used for hemoglobin determinations?

Cyanmethemoglobin

All of the following are roles of a sentinel microbiology laboratory, EXCEPT?

Determine that a bioterrorist event has occurred.

Acute obstructive epiglottitis, both in adults and children, is most commonly caused by a specific bacterial species that can be recovered on chocolate agar and requires both hematin and NAD. The most likely identification is:

Haemophilus influenzae

Cells that stain positive with acid phosphatase, and are NOT inhibited with tartrate, are characteristically seen in:

Hairy cell leukemia

All of the following cellular antigens are important to an immunohematologist EXCEPT:

Haptens

Which of the following culture media should be used to attempt to recover mycobacteria from a clinical specimen?

Lowenstein-Jensen (LJ) medium

Patients may sometimes develop antibodies to which antigen without having a red cell stimulation?

Lua

A gram-negative diplococcus is isolated from a throat culture. The isolate grows on Thayer-Martin agar, produces acid from maltose, lactose, and glucose and is ONPG positive. What is the most likely identification of this isolate?

Neisseria lactamica

What procedure utilizes leukapheresis to collect the buffy coat from whole blood?

Photopheresis

Which animal was involved in the reassortment of influenza A viruses that led to the 2009 novel influenza A H1N1 virus?

Pigs

To enhance recovery of bacteria, proper inoculation of broth media may be accomplished by:

Placing a few drops of liquid specimen directly into the tube of broth

Which of the following is an example of a macroscopic urinalysis abnormality?

Positive protein

Which of the following macroscopic findings is MOST consistent with the microscopic finding of casts?

Positive protein

In which of the following conditions would one see Pappenheimer bodies on the peripheral blood smear?

Splenectomies

A 6-year-old girl was seen in the emergency department with pain and tenderness in her lower right quadrant. A CBC showed a high white blood cell count with increased segmented neutrophils and bands. The child was admitted to surgery with a diagnosis of appendicitis. During surgery, the appendix appeared normal, but an enlarged node was removed and cultured. A diagnosis of mesenteric lymphadenitis was made. Gram-negative rods were isolated. Which of the following biochemical results are most likely produced by the suspect organism?

TSIA acid over acid, H2S negative, lysine negative, indole variable, urea positive

The following are organisms that are associated with their appropriate diagnostic specimens with the exception of:

Trichomonas tenax- vaginal secretions

Identify the genus, species and stage for the organism seen in this image.

Trichuris trichiura egg

Toxic granulation is seen most frequently in:

bacterial infections

Identify the nucleated cell in the image.

monocyte

How are Barr bodies usually classified?

non-pathological

Which restriction enzyme class does not have methylation activity?

Type II

What is generally accepted as the lower threshold value for semen pH from fertile males?

7.2

When performing a point-of-care test for glucose, what is the reference range for glucose?

74-100 mg/dL

The A1 subgroup represents approximately what percentage of group A individuals?

75 - 80%

Which of the following organisms is most likely to be associated with gas gangrene:

Clostridium perfringens

Which of the following organisms produces a double zone of hemolysis on blood agar?

Clostridium perfringens

What are function(s) of the platelet?

Clotting, plug formation, release of serotonin, and repair of the injured tissues.

Tricyclic Antidepressants (TCAs) are used to treat insomnia, depression, extreme apathy, and loss of libido. All of the following are TCAs, EXCEPT?

Clozapine

Parasites that belong to the category Sporozoa are also known as:

Coccidia

The Clinitest® reaction is based on which of the following principles?

Copper reduction

A patient is suspected of having Cushing's syndrome. His doctor is ordering a cortisol level. Which of the following are true about the diurnal variation of cortisol levels in blood?

Cortisol levels are higher in the morning.

An uncommon laboratory isolate, the colonies on blood agar, as seen in the top upper frame of the composite image were cultured from the throat swab of a 10 year old girl presenting with a red throat and mucosal pseudomembrane. These tiny gray-white, non hemolytic colonies are non-descriptive. Distinctive are the black pigmented colonies as observed on Tinsdale agar (lower image in the top composite photograph). Illustrated in the lower composite photomicrograph (upper image) are slender gram positive bacilli. Bacilli with meta-chromatic granules are observed in a methylene blue stained microscopic mount (lower image). From these observations, select from the multiple choices the presumptive identification of this isolate

Corynebacterium diphtheriae

illustrated in the upper photograph are tiny, gray-white, non-hemolytic colonies growing on the surface of blood agar. Recovery is most often from infections in immune-suppressed hospitalized patients. The Gram stain shows Gram-positive bacilli arranged in a "diphtheroid" pattern. Characteristic for making a species identification of this isolate are negative reactions for all enzymes. From these observations, select the presumptive identification of this isolate.

Corynebacterium jeikeium

Which one of the following serum constituents is increased following strenuous exercise?

Creatinine

Which of the following tests would be useful in the assessment of glomerular filtration:

Creatinine clearance

What drug may be used to decrease iron levels in patients with iron overload?

Desferrioxamine

What is the most likely interpretation of the amplification plot below?

Presence of Bordetella parapertussis

What accelerates the rate of lipase activity in the body?

Presence of bile salt

When preparing a slide for morphologic examination, what is the reason for adding a drop of albumin to the cerebrospinal fluid (CSF) sample prior to cytocentrifugation?

Preserve the integrity of the cells.

A solution of gamma globulins containing anti-Rh(D) is given to an Rh(D) negative mother to:

Prevent fetal cells from initially sensitizing the mother

Rh Immune Globulin (RhIG) is used to:

Prevent the formation of anti-D

What is the focus of both Lean principles and the Six Sigma methodology?

Process improvement

All of the following characteristics describe T lymphocytes EXCEPT?

Production of surface immunoglobulins

Identify the leukocyte seen in this illustration:

Promyelocyte

Which potent inhibitor of platelet aggregation is released by endothelial cells?

Prostacyclin

All of the following formed elements may be present in urine sediment and can be identified microscopically, EXCEPT:

Protein

Which of the following analytes can be detected by the urine reagent strip in a point-of-care setting?

Protein, glucose, leukocytes, and bilirubin

In the image to the right, the letter B represents what type of endospore placement?

central

The organisms Klebsiella pneumoniae, Enterobacter cloacae, and Escherichia coli can produce carbapenemases due to which of the following genes?

blaKPC

The oxidase test is used to presumptively identify which of these organisms?

neisseria

The appropriate media for the isolation of anaerobic bacteria from any source would normally include all of the following except:

MacConkey agar

A manual white blood cell count was performed by the hematology technologist. The cell counts for each of two sides was 38 and 42 respectively. All nine large squares were counted on each side. The dilution for this kit was pre-measured at 1:10. What should the technologist report as the white cell count?

0.44 x 109/L

The concentration of sodium chloride in an isotonic solution is:

0.85 %

Determine the molarity of a solution of HCl that is contained in a Class A 1-L volumetric flask filled to the calibration mark. The content label reads 32 g HCl. The gmw of HCl=36.46 g/mol.

0.88 mol/L ((32 g HCl / L) X (1 mol/ 36.46 g HCl) = 0.88 mol/L)

If present, how many Howell-Jolly bodies are usually seen within a given erythrocyte?

1

With regard to blood cultures, which blood to broth ratio is most conducive to growth?

1 : 10

All of the terms below are a measure of central tendency, EXCEPT:

Molarity

Whole blood or red blood cell units will be shipped from the American Red Cross to the transfusion facility. At what temperature should the units be kept during transportation?

1-10 °C

The laboratory workplace includes many hours of computer use. The suggested time for a break after 30 minutes of computer use is:

1-2 minute break

A 45-year-old male of average height and weight had a serum creatinine of 1.5 mg/dL and urine creatinine was 120 mg/dL; the total volume of urine collected over a 24-hour period was 1,800 mL. Calculate the creatinine clearance for this patient in mL/min.

100

Which of the following represents the reference range for hemoglobin for an adult woman?

12 - 16 g/dL

Which of the following GFR (glomerular filtration rate) values is considered to be in renal failure?

14 mL/min

A unit of red blood cells that was collected on 15 June 2009 and frozen with glycerol at -80° C on 20 June 2009 will expire on what date?

15 June 2019

What is the lower reference limit for sperm concentration according to the World Health Organization, WHO Laboratory Manual for the Examination and Processing of Human Semen, WHO press, Geneva, Switzerland, 2010?

15 x 106 spermatozoa/mL

Approximately what percent of the Caucasian population is Rh negative?

15%

A 1:10 dilution is made on a CSF sample. Five squares on each side of the hemacytometer are counted for a total of 10 squares and a total of 150 cells are recorded. What is the count per microliter?

1500

The formula for conversion of hemoglobin A1C (HbA1C) to glucose in mg/dL is eAG = (28.7 x A1C) - 46.7. The HbA1C measured on a patient is reported as 7.5%. What would be reported as the estimated average glucose (eAG) for this % A1C (rounded to the nearest whole number)?

169 mg/dL

After an acute myocardial infarction (AMI), myoglobin levels will rise within 3 hours and peak at 8-12 hours. When will they return to normal?

18-30 hours

Consider the following regarding infection with Mycobacterium tuberculosis. Each response is correct EXCEPT:

A positive purified protein derivative (PPD) test always indicates the presence of active tuberculosis disease.

Significant risk factors for potential development of graft-versus-host disease (GVHD) include the following factors except:

A strong ability of the host to destroy foreign lymphocytes

Calculate LDL cholesterol concentration if total cholesterol is 280 mg/dL, triglyceride is 100 mg/dL, and the HDL cholesterol is 40.0 mg/dL.

220 mg/dL

After frozen RBC's have been thawed and washed, they must be used within how many hours?

24

FFP that has been thawed at 30 - 37°C and maintained at 1 - 6°C must be transfused within ___________ after it has been thawed.

24 hours

How soon must granulocyte concentrates be administered after donation?

24 hours

How many grams of NaOH is required to prepare 500 ml of a 5% (w/v) NaOH solution?

25 grams

Which of the following fresh frozen plasma ABO types would be suitable for transfusion to an AB negative patient?

AB negative and AB positive only.

The direct antiglobulin test (DAT) is most unreliable when diagnosing hemolytic disease of the fetus and newborn due to which blood group system?

ABO

Units of A-negative, B-negative, and O-negative red blood cells are shipped to your transfusion service. What testing MUST be performed by your facility before placing these units into your inventory?

ABO and Rh

Which of the following is the most important factor in determining the immediate outcomes of kidney transplants?

ABO compatibility

The presence of hemolysis in a post-transfusion blood sample is best associated with which of the following?

ABO incompatibility

What is the cause of neonatal alloimmune thrombocytopenia (NAIT)?

Destruction of a neonate's platelets by alloantibodies crossing the placenta from the mother.

A spinal fluid that is slightly hazy is briefly examined microscopically. The technologist performing the count decides to make a 1:10 dilution using 30 µL of sample. What volume of diluent should be used?

270 µL

What is the normal myeloid to erythroid (M:E) ratio in the bone marrow?

2:1 - 4:1

The normal Myeloid-to-erythroid ratio (M:E ratio) ranges from:

2:1 to 4:1

Antibody identification interpretations would be considered correct 95% of the time or have a P value of 0.05 (5% probability that the result is due to chance) if you have:

3 positive reactions to rule in an antibody and 3 negative reactions to rule out an antibody

Troponin I is used frequently to assess acute myocardial infarctions (AMI). If a patient has experienced an AMI, at what point (approximately) will the troponin I begin to increase and how long will it stay increased?

3-12 hours, 7-14 days

Classification of acute leukemia in the United States relies on two main systems, the FAB (French American British) and the WHO (World Health Organization) classifications. In order to diagnose acute leukemia, the FAB system requires ______% of non-erythroid blasts to be present in the bone marrow/peripheral blood, while the WHO system requires the presence of ________% non-erythroid blasts.

30%, 20%

The recommended storage temperature for the preservation of activity for MOST enzymes is:

4ºC

What is the composition of basophilic stippling?

Aggregation of ribosomes

Mycobacterium tuberculosis can be spread from person to person via which transmission method?

Airborne

How are the bacterial organisms that cause tuberculosis (Mycobacterium tuberculosis) primarily transmitted?

Airborne droplets/aerosols

Donated red blood cells that contain the anticoagulant CPDA-1 (citrate-phosphate-dextrose-adenine) may be stored up to how many days?

35 days

Which of the following is the proper storage temperature for whole blood?

4 degrees Celsius

An Rh negative mother has just given birth to an Rh positive baby after 18 hours of strenuous labor. Her rosette test was positive. Upon performing the Kleihauer-Betke stain procedure, the percentage of fetal cells is found to be 1.9%. The mother's total blood volume is 5,000 mL. What dose of Rh Ig (RhoGam) should be administered to the mother?

4 vials

Calculate the red cell indices from the following set of patient data: RBC count = 3.19 x 1012/L Hemoglobin = 11.3 g/dL Hematocrit = 35%

MCV = 110 fL, MCH = 35 pg, MCHC = 32 g/dL

What is the ideal fragment length for NGS library prep?

400 bp

What is the total magnification produced when using a 10X ocular lens and a 40X objective lens on a Bright Field light microscope?

400x

The appropriate magnification for a manual RBC cell count using a hemocytometer is which of the following?

40X (Dry)

The anaerobe growing on the surface of the blood agar plate, the toxins of which cause a form of necrotizing enteritis known as Darmbrand ("fire bowels") in Germany or "Pig bel" in New Guinea, is:

Clostridium perfringens

Within what period of time after performance of the tuberculin skin test (TST) should the result be read?

48-72 hours

When performing a manual white blood cell count, 236 cells were counted on side number one of the hemacytometer, 224 cells on side number two. A 1/20 dilution of 1% HC1 was used, and the area counted on each side was 4 sq. mm. Twenty-five nucleated red blood cells in the differential were observed per 100 white blood cells. What is the CORRECT WBC/µL?

9,200

What is the corrected white blood cell count if the WBC is 14,460, and there were 47 nucleated red blood cells per 100 white blood cells noted on the differential count?

9,837s (Corrected WBC count = uncorrected WBC count x (100 / NRBCs +100))

What is the maximum number of white blood cells allowed in a leukocyte-reduced unit of red blood cells?

5 x 10^6

What percentage of dietary iron is normally absorbed daily?

5% to 15%

The primary antibody response takes an average of how many days?

5-10 days

How long does it take for plasma concentrations to reach steady state when a patient's dose is given at intervals of the drug's half-life?

5-7 half-lives

A CSF glucose value is 62 mg/dL. What would you estimate the serum glucose?

93 mg/dL

As defined by a Gaussian distribution curve, what percentage of values would be expected to fall within two standard deviations of the mean?

95%

In a normal distribution of results, the mean value +/- 2 SDs will include what percentage of the population?

95.5%

The following organism is an anaerobic organism that produces spores and a double-zone of beta-hemolysis.

Clostridium perfringens

If an Rh group (DCe/dce) man marries an Rh group (dce/dce) woman, what is the probability that their offspring will be D negative?

50%

If parents have the blood group genotypes AA and BO, what is the possibility of having a child with a blood type of A?

50%

A mean value of 100 mg/dL and a standard deviation of 1.8 mg/dL were obtained from a set of glucose measurements on a control solution. Based on a 95% confidence interval (in mg/dL), what would be the calculated 2 SD upper and lower limits for this control?

96.4 to 103.6

The De Ritis ratio is the calculation of AST/ALT and is used for evaluating liver disease. What should the ratio be when ethanol-related liver disease is expected?

>2.0

Pleural fluid was sent to the laboratory for a WBC count. The specimen was NOT diluted.WBCs in 5 large squares were counted in each side of the hemacytometer.The number of WBCs counted were as follows:Top counting chamber: 35 cells total (in 5 large squares)Bottom counting chamber: 33 cells total (in 5 large squares)Based on the information provided above, calculate the total WBC count.

68 cells/mm3

In order to prevent a loss of viability in platelet concentrates during storage, the pH must be maintained at or above what level?

6.2

Blood is diluted at 1:100 and charged on the hemocytometer counting chambers on both sides. If the average platelet count from a counting chamber in one mm2 (one large square) is 68, what is the calculated platelet count?

68,000/mm3

Donor and recipient blood samples must be kept for at least how long after transfusion?

7 days

Which of the following tests are affected by the pneumatic tube system transport?

Acid phosphatase

The positive oxidase reaction as shown in the image (yellow arrow) rules out which of the following "look-alike" bacteria that resemble N. gonorrhoeae?

Acinetobacter baumannii

illustrated in the upper composite photograph are non-hemolytic, smooth, entire non-pigmented white colonies growing on blood agar. Human isolates are most commonly from nosocomial infections related to endotracheal tubes and tracheostomies, with peritonitis in patients receiving peritoneal dialysis. Distinctly light lavender-pink pigmented colonies are observed growing on MacConkey agar. Tiny coccobacilli simulating those of Neisseria species are observed in gram stains as illustrated in the lower photomicrograph. Other identifying features include the oxidative production of acids from most carbohydrates (saccharolytic) and a negative oxidase test. From these observations, select from the multiple choices the presumptive identification of this isolate.

Acinetobacter baumannii

When would unexpected positive reactions be encountered during ABO forward typing?

Acquired B antigen

Which Gram positive bacilli will show a positive result for urease?

Actinomyces naeslundii

Both hemoglobinuria and hemoglobinemia will be present in what type of reactions?

Acute hemolytic transfusion reactions

Given the following hepatitis B serology results, what is the immune status of the patient? HBsAg: positive HBeAg: positive Anti-HBc IgM: positive Anti-HBs: negative

Acute infection

A 27-year-old man with Down syndrome has the following CBC results which are MOST suggestive of what condition? WBC: 27 x 103/µL HGB: 8.5 g/dl HCT: 25.0% PLT: 30 x 109/L Differential blood smear results: 8% segmented neutrophils 25% lymphocytes 67% TdT-positive blasts

Acute lymphocytic leukemia

The name of a reaction between a single antigenic determinant and an individual combining site is:

Affinity

The arrangement of the erythrocytes shown by the blue arrows in this peripheral smear should be reported out as ____________________ .

Agglutination

What is the composition of the granules associated with basophilic stippling?

Aggregates of ribosomes

Which beta thalassemia shows no anemia and may not be detected using hemoglobin electrophoresis?

Beta thalassemia minima

These test results are most consistent with which of the following types of beta thalassemia? TestPatient ResultReference Intervals (Adult female)Red blood cell (RBC) count5.6 x 1012/L4.1 - 5.1 x 1012/LHemoglobin (Hb)11.2 g/dL12.3 - 15.3 g/dLHematocrit (HCT)34.5%35.9 - 44.6%RDW14.0<14.5 RBC morphology- Microcytes, basophilic stippling, codocytes (target) present.

Beta thalassemia minor

Which of the following globin genes are found on Chromosome 11?

Beta, gamma, epsilon

The optimum time to collect a urine sample to maximize recovery of the ovum illustrated in this iodine-stained mount of urine sediment is:

Between Noon and 3:00 P.M

In the formalin-ethyl acetate sedimentation procedure, four layers are present in the tube following centrifugation. Which layer contains the formalin?

Between the bottom layer and the debris/fat layer

Glycine-HCL/EDTA treatment of red cells can destroy which of the following antigens, allowing for confirmation of a suspected antibody and detecting additional antibodies?

Bg and Kell

MOST of the carbon dioxide present in the blood is in the form of:

Bicarbonate

Given the following data, what is the most likely disease state? Total bili = 4.0 (0.5-1.5) Direct bili = 3.0 (0.2-0.5) AST = 75 (2-25) ALT = 118 (2-23) LD = 220 (< 140) ALP = 841 (28-68) GGT =150 (21-40 U)

Biliary duct obstruction

All of the following serological test results may occur in a patient experiencing a hemolytic transfusion reaction due to the presence of a clinically significant Rh antibody such as anti-c EXCEPT:

Binding complement.

Which of the following choices best describes the primary function of antibodies?

Binds with antigen

Which of the following methods requires the chemical process of amination to permit the differential readout of DNA methylation?

Bisulfite sequencing

The suspicious form pictured here in a red cell (lacking Schüffner dots) is responsible for which of the following conditions?

Black water fever

This image represents parasites belonging to which of the following groups?

Blood Nematodes

The impedance principle shown in this illustration is best described by the following statement:

Blood cells or other particles are counted and sized based on changes in electrical resistance as they pass between two electrodes.

All of the following factors would cause an increase erythrocyte sedimentation rate (ESR) values EXCEPT?

Blood drawn into sodium citrate anticoagulant

To assess drug concentrations during the trough phase:

Blood should be drawn immediately before the next dose is given.

Which of these methods are used to identify prior exposure to Mycobacterium tuberculosis?

Blood tests known as interferon-gamma release assays.

When a group O patient's serum is not compatible with donor O red blood cells and the auto control is negative, what is the MOST likely cause?

Bombay phenotype

Which of the following statements is correct regarding bone marrow evaluation?

Bone marrow cellularity is highest in young children compared to adults.

Illustrated in this photograph is a positive slide coagulase test (Image B) compared to a negative control (Image A). The clumping of the bacterial cells, as seen in Image B, results from the bacterial production of:

Bound coagulase

All of the following are sources of serum alkaline phosphatase EXCEPT:

Brain

All of the following are commercial methodologies that Chlamydia and Neisseria identification have been based on, EXCEPT?

Branched chain DNA technology

The recovery of Mycoplasma hominis from the genital tract of females has been associated with all of the following clinical syndromes EXCEPT:

Bronchitis

Which Brucella species has positive H2S production and may or may not need CO2 for grow?

Brucella abortus

This species is the most commonly reported laboratory-acquired bacterial infection and must be handled in class II or higher biological safety cabinet. They are small gram-negative coccobacilli that are strict aerobes and will grow on Sheep Blood Agar and Chocolate agar. They are nonmotile, catalase, urease, and nitrate positive and most strains are oxidase positive. From the choices below, please select the organism matching the microscopic and biochemical reactions listed:

Brucella species

Tiny Gram-negative coccobacilli were isolated on chocolate agar from a wound on a man who had been hunting wild boar. The microbiologist performed the following test: OxidasePositive UreasePositive MotilityNonmotile Colony morphologySmall, nonhemolytic Which of the following species is most probable?

Brucella species

Which of the following phenotypes is most indicative of a natural killer (NK) cell?

CD2+ CD3- CD11b+ CD16+

In HbSS blood, an increased amount of which of the following surface antigens on young sickle cells (reticulocytes) may allow platelets to form a bridge between the reticulocytes and endothelial cells, ultimately leading to vaso-occlusion?

CD36

Which of the following genotypes is found with the highest frequency in the Caucasian population?

CDe/ce

Persons who have received a dura mater transplant are not eligible to donate blood and are permanently deferred. This is due to an increased risk of which of the following?

CJD or vCJD

Suppose a patient with stiffness in her fingers has a positive antinuclear antibody (ANA) test with a centromere pattern at a 1:1280 titer. What is the most likely diagnosis?

CREST syndrome

Identify the inclusion indicated by the arrow:

Cabot ring

Image C represents which of the following?

Cabot rings

The erythrocyte inclusions, which are thin, red-purple staining strands, usually found in figure-eight shapes, are called:

Cabot rings

Which substance is used in the Jendrassik-Grof method to accelerate the reaction of unconjugated bilirubin with the diazo reagent?

Caffeine-benzoate

EDTA and Sodium citrate combine with _________ to inhibit coagulation in blood samples.

Calcium

All of the following normal urine crystals are most often found in a urine specimen that has a pH > 7 EXCEPT:

Calcium oxalate

What is the identification of these crystals seen in a urine with an acid pH?

Calcium oxalate

Identify the urine sediment elements indicated by the arrows in this microscopic field.

Calcium oxalate crystals

Which of the following would be the most characteristic finding in synovial fluid in a case of pseudogout?

Calcium pyrophosphate crystals

All of the following are types of acid-fast stains, EXCEPT:

Calcofluor white

Which stain is a colorless dye that binds to chitin in fungi and fluoresces with ultraviolet light?

Calcofluor white

Which of the following stains is commonly used to stain chitin in fungal cell walls?

Calcoflur-white stain

The principle behind point-of-care devices to measure hematocrit is:

Calculating the conductance of the blood sample and correcting for temperature, conductivity of the plasma, and size of the fluid segment

Different species of Neisseria can be differentiated from each other by:

Carbohydrate utilization

Which of the following consequences of severe hemolytic disease of the fetus and newborn (HDFN) is most associated with neonatal death before or shortly after birth?

Cardiac failure

Beta hemolytic colonies grew from the blood culture bottle after 18 hours of incubation (see image). If the gram stain shows gram-positive cocci, which of the following tests would be helpful with making a preliminary identification?

Catalase

Which of the following is a TRUE statement concerning the Category A agents of bioterrorism?

Category A agents cause high mortality and the potential for major public health impact.

For an isolate of Escherichia coli, Klebsiella, or Proteus species testing positive for ESBL (Extended Spectrum Beta Lactamases), all of the following antibiotics should be reported as resistant except:

Cefoxitin

Which of the white cells indicated by the arrows in the image is normally the most numerous cell type found in the peripheral blood of an otherwise healthy adult?

Cell "a" = segmented neutrophil

Which one of the following statements concerning phagocytosis is FALSE?

Cells are only capable of phagocytizing bacteria.

The laboratory receives a synovial fluid for acid-fast culture. How should the tech proceed with this culture?

Centrifuge the specimen and directly inoculate the culture

What antinuclear antibody (ANA) staining pattern is demonstrated in this microscopic, fluorescent test image?

Centromere

Which one of the following drugs/drug classes is the MOST COMMON cause of drug-induced immune hemolytic anemia?

Cephalosporins

CSF lymphocytosis is associated with all of the following EXCEPT:

Cerebral abscess

What is the optimal specimen type to diagnose a Naegleria fowleri infection?

Cerebrospinal fluid (CSF) sample

In this photomicrograph, circulating trypomastigotes were observed on a blood smear and were identified as Trypanosoma cruzi. The infection rate in the community has increased and was linked to a transfusion-transmitted disease. What disease does this parasite cause?

Chagas disease

An aliquot of AS-1 red blood cells is being prepared from an intact packed cell unit using a sterile connection device.During the process of preparing an aliquot, the sterile device fails and blood drips onto the counter from the product tubing. What should be done with the primary unit?

Change the expiration date to 24 hours

What is the FIRST thing blood bank staff should do when investigating a transfusion reaction?

Check for clerical errors in all steps of the pre- and post processes.

What is the most likely clinical condition in which the cell in the image A would be present in increased numbers?

Chediak-Higashi

The inclusions that are seen in the white cell indicated by the arrow in this image are characteristic of which of the following conditions?

Chediak-Higashi syndrome

Which disease/condition can be indicated by the morphology displayed in the white blood cell to the right?

Chediak-Higashi syndrome

A 50-year-old male was rushed to the hospital after experiencing tremendous fluid loss due to severe diarrhea. History revealed that the patient was diagnosed with AIDS 6 months ago. The doctor ordered a battery of tests including a stool for parasite examination. Since the sample was properly labeled indicating that the patient was immunocompromised, the laboratory performed both the standard processing procedures and a modified acid-fast (mod AFB) stain. The mod AFB stain revealed this suspicious form seen in the image which measured 4 µm. This patient is most likely infected with:

Cryptosporidium parvum

The most highly specific indicator for rheumatoid arthritis is:

Cyclic citrullinated peptide antibodies (CCP antibodies)

Identify the urine sediment element shown by the arrow.

Cylindroid

What is the infective stage for the intestinal amebae?

Cyst

Which of the following is the rigid and nonmotile diagnostic stage of amebae that is generally found in a formed stool?

Cyst

Which of the following best describes a resting lymphocyte?

Cytoplasm is light blue and nucleus is about the size of an erythrocyte.

In which of the following condition(s) would an increased serum osmolality be an expected finding?

Dehydration and Diabetes insipidus

Antibodies to the Kidd blood group system are MOST commonly associated with:

Delayed hemolytic transfusion reactions

Which action can cause of laboratory-related error?

Delayed transport of laboratory specimens

The hemoglobin electrophoresis pattern for patient #2 shows an increase in Hb F and a decrease in both Hb A and Hb A2. These results correlate best with which of these thalassemias?

Delta-beta thalassemia minor

Which of the following is the most common method for diagnosing malaria?

Demonstration of the organism in peripheral blood

At a medical examination, a 50-year-old Caucasian male expressed concern regarding diabetes. There is a history of type 2 diabetes, hypertension, and cardiovascular disease in his family. He has gained a few pounds each year, and his physician notes abdominal obesity. His physician orders laboratory tests to evaluate his risk of cardiovascular disease. Vital Signs and Pertinent Laboratory Results: Blood Pressure: 128/82 mm HgWeight: 230 lbsHeight: 5' 11''Calculated BMI: 32.1Waist Circumference: 45 inches Fasting Blood Glucose: 120 mg/dLTriglycerides: 170 mg/dLHDL-C: 42 mg/dL Which one of the following statements regarding this patient is true if the physician uses the American Heart Association (AHA) and National Heart, Lung, Blood Institute (NHLBI) guidelines for metabolic syndrome evaluation?

Diagnosis of metabolic syndrome; waist circumference, triglyceride level, and fasting blood glucose meet the criteria

The most common methods for measuring bilirubin are based on the reaction of bilirubin with:

Diazo reagent

Which of the following types of automated chemistry analyzers allows for each specimen and accompanying reagents to have their own space?

Discrete analyzer

What action should be taken if a large clot is noticed in a red blood cell unit while the product is being prepared for release to the patient?

Do not issue the product.

What is considered the definition of allogeneic blood?

Donated by a donor for a recipient other than the donor to use

The major crossmatch is performed using:

Donor's red cells and recipient's serum or plasma

If a pipette is labeled (TC) "to contain" you would do the following:

Drain contents then rinse out with diluting fluid

Observe the peripheral blood smear image to the right. Several types of poikilocytes are seen. All of the following are seen in the image EXCEPT?

Drepanocyte

Thin-layer chromatography is particularly useful as a tool in the identification of:

Drugs

A patient experiences a mild allergic reaction to a transfusion, including urticaria, erythema (skin redness), and itching. What is the most likely source of the allergen?

Drugs or food consumed by the blood donor

For the diagnosis of Systemic Lupus Erythematosus (SLE) which of the following test principles provides the highest specificity?

ELISA

Humans serve as accidental (incidental) hosts in the life cycle of which one of these parasites?

Echinococcus granulosus

Illustrated in this photograph is a close-in view of colonies growing on the surface of xylose lysine deoxycholate (XLD) agar. The bacterial species most likely producing the colonies shown here is:

Edwardsiella tarda

The morphology of this cyst resembles Entamoeba histolytica, but lacks ingested red blood cells in its trophozoite form. What is this organism?

Entamoeba hartmanni

What is this trichrome-stained suspicious form that was found in a stool sample, and measures 15 µm?

Entamoeba histolytica

The bile esculin test has been used to determine if an organism can hydrolyze esculin. Esculin fluoresces, so if it is not hydrolyzed, the tube will fluoresce (bucket B), but if it is hydrolyzed, no fluorescence is seen (bucket A). Which of the following organisms hydrolyzes esculin?

Enterococcus faecalis

Illustrated in the top photograph is a 4-day growth of silky, light gray-white to yellow colonies grown on Sabouraud's dextrose agar incubated at 30° C, recovered from an "athlete's foot" infection (tinea pedis) of a young football player. Note the projection of delicate hyphae to the periphery of the colonies. The identification can be made by observing the conidia formations in the lactophenol blue-stained mount prepared from the surface of one of the colonies. With these observations, select the species identification from the multiple choices.

Epidermophyton floccosum

When performing a transfusion reaction investigation, what is the clerical check used to detect?

Errors that may cause an ABO-incompatibility due to patient or donor unit mix up

Ninety-nine percent of lead absorbed in the body is taken up by what after absorption?

Erythrocytes

All of the following are indicators of sepsis, EXCEPT?

Erythrocytosis

A D-test is performed on gram-positive cocci to determine inducible resistance to clindamycin when the isolate is sensitive to clindamycin and resistant to which antibiotic?

Erythromycin

Which porphyria has a defect in the ferrochetalase enzyme within the RBC, causing redness, edema and burning when exposed to light?

Erythropoietic protoporphyria

Which of the following is a common cause of urinary tract infections?

Escherichia coli

Which of the following specimen processing techniques is based on the principle that parasites are heavier than sample debris and will be present in the sediment after being processed?

Ethyl acetate concentration

A 62-year-old man drank an unknown liquid in a suicide attempt, and his urine contained crystals similar in shape to those shown in this image. This man MOST likely ingested:

Ethylene glycol

When an automated or semi-automated method is used to read urine reagent strips, quality control testing must be performed at least:

Every day of patient testing and when a new bottle is opened.

Which pair of PCR stage and temperature is correct?

Extension at 72 °C

Before DNA can be amplified, it must first be isolated via

Extraction

Which FAB designation is called the "true" monocytic leukemia and is characterized by monoblasts, promonocytes, and monocytes?

FAB M5

An increased D-Dimer can be associated with all of the following EXCEPT:

Factor Deficiency

Hemolysis may be caused by all of the following EXCEPT:

Failing to invert the blood tube after collecting to mix it with anticoagulant.

Concerning antigen-antibody reactions, the prozone effect causes which of the following results?

False negative

A urine specimen is received in the laboratory late in the afternoon. The specimen was collected early in the morning and was accidentally left in bright sunlight and at room temperature on a counter in the outpatient clinic. The test order is for urine bilirubin screening. Which of the following could occur as a result of the storage conditions?

False-negative test result

Illustrated in this high-power microscopic view is a large spherical ovum measuring 130 µm that was found in a stool specimen. These ova often have brown staining that suggests passage through a biliary duct on its way to the large intestine. Based on the appearance of this ovum, select the correct presumptive identification.

Fasciola hepatica

A 55-year-old female, who had recently returned from an extensive trip to China, presented to her physician complaining of diarrhea and abdominal cramps. A complete blood count (CBC), comprehensive metabolic panel (CMP), and stool for both culture and parasite examination (O & P) were ordered. The CBC revealed pronounced eosinophilia. The CMP and stool culture were unremarkable. The O & P revealed this suspicious form that measured approximately 140 µm by 80 µm. This patient is most likely infected with:

Fasciola/Fasciolopsis eggs

Illustrated in this image is a large adult trematode averaging 3 - 7 cm. This fluke resides primarily in the small intestine. Characteristic is the rounded cephalic end, the convoluted uterus in the anterior half of the fluke, and delicate branching testes in the posterior half. Humans become infected by ingesting metacercaria contaminated water vegetables. Select the presumptive identification.

Fasciolopsis/Fasciola species

The following urine test results were obtained from a 6-month-old African American infant who experiences vomiting and diarrhea after milk ingestion and has failed to gain weight: pH: 5.0 Protein: Negative Glucose: Negative Ketones: Negative Blood: Negative Bilirubin: Negative Nitrite: Negative Urobilinogen: 0.1 EU/dL Clinitest: 2+ These results are clinically significant in which of the following disorders?

Galactosemia

Diseases associated with a dysfunction of polymorphonuclear neutrophils (PMNs) include all of the following with the exception of:

Gaucher 's Disease

All of the following represent a 2 week temporary deferral from donating blood or blood products EXCEPT:

German measles (rubella) vaccine

A 35-year-old male presented to the local clinic complaining of abdominal cramps, severe diarrhea, and intestinal gas. A foul-smelling and light-colored stool specimen was collected and sent to the laboratory for parasite examination. This form was recovered from the stool concentrate and measured 10 µm by 12 µm. The patient is infected with which one of the following organisms?

Giardia duodenalis (formerly lamblia)

Which of the following conditions will cause a positive reagent strip blood result due to hematuria?

Glomerulonephritis

All of the following parameters are generally considered to be a part of seminal fluid analysis, EXCEPT?

Glucose concentration

The reactions in the tubes illustrated in this exercise (reading from left to right: TSI, Pseudosel, OF glucose closed, and OF glucose open) indicate that the bacterial species in question is:

Glucose oxidizer

A Basic Metabolic Panel (BMP) consists of which group of tests?

Glucose, BUN, Creatinine, Sodium, Potassium, Chloride, CO2, Calcium

Which test is performed to assess the average plasma glucose level that an individual maintained during the previous 10 to 12 weeks?

Glycosylated hemoglobin

Which analyte is MOST useful for monitoring the long-term (~12 weeks) stability of blood glucose?

Glycosylated hemoglobin (HbA1C)

The presence of Gram-negative intracellular diplococci in a genital specimen from a male is diagnostic of which disease?

Gonorrhea

Which of the following conditions is associated with elevated serum uric acid levels:

Gout

Based on the image to the right, the issued gram stain report should read, "Many WBC with..."

Gram positive cocci in pairs.

Observation of each of the following characteristics is helpful in differentiating between Nocardia species and Mycobacterium species EXCEPT:

Gram positive staining

Which of the following categories includes mature neutrophils, eosinophils, and basophils?

Granulocytes

Which of the following statements represents alpha hemolysis on 5% sheep blood agar?

Green area around bacteria; incomplete hemolysis

Asaccharolytic use of glucose would be indicated by which of the following reactions in OF glucose tubes?

Green in both tubes with a little blue at the top of the open tube

Which of the following organisms would show as negative for the testing shown in the image?

H. Influenzae (The image shows delta-aminolevulinic acid disks, which are used in the porphyrin test. The porphyrin test is a method for determining the heme-producing species of Haemophilus. The principle of the test is the ability of the organism to convert the substrate (delta-aminolevulinic acid) into porphyrins or porphobilinogen. Both of these are intermediates during the synthesis of heme or X factor. Porphyrins fluoresce a reddish-orange color under ultraviolet light. If the test is negative, the organism cannot synthesize heme (Factor X) and thus would require it for growth. If the test is positive, the organism can synthesize heme (Factor X) and does not require heme as a separate growth factor. Haemophilus influenza requires both X (heme) and V factor (NAD) as separate factors in order to grow, thus the porphyrin test would be negative for H. influenzae.)

Which is the first marker (antigen or antibody) which will become positive after exposure to Hepatitis B?

HBsAg

What is the expected serological response in an immunocompetent patient who has been vaccinated for hepatitis B?

HBsAg (neg), Anti-HBc(neg), anti-HBs (pos)

Which gene loses function in hemochromatosis?

HFE

The appearance of small colonies, growing adjacent to large Staphylococcus aureus colonies on the surface of the sheep blood agar plate indicates the need for Factor V for growth (upper photograph). This is confirmed by the X and V Factor Test in which growth is observed only around the V impregnated strip, but not around the X filter paper disc. Factor X is included in the agar medium. With these observations, select from the multiple choices the name of this isolate.

Haemophilus parainfluenzae

The accuracy of an immunoassay is its ability to discriminate between results that are true positive and results that are true negative. Two parameters of test accuracy are specificity and sensitivity. Which of these statements apply to an immunoassay with low sensitivity?

Has many false-negative results

In order to avoid repeating pretransfusion testing on a neonate during one hospital admission, all of the following must be true, EXCEPT?

Has only received formula for nourishment

All of the listed disorders are organ non-specific autoimmune diseases EXCEPT:

Hashimoto's disease

A microbiologist was performing Shigella serotyping on a probable Shigella isolate and the organism did not serotype. The microbiologist was adamant that the organism was a Shigella species because it was lactose negative on Hekton-Enteric agar and non-motile in motility agar. What should the microbiologist do to ensure that false negative testing did not occur?

Heat specimen for 15 minutes and repeat testing

Which of the following inclusions, marked by the arrow, is seen on this smear made with a supravital stain?

Heinz bodies

If the red cell count is known, which of the following MUST also be known in order to calculate the mean corpuscular volume (MCV)?

Hematocrit

The percentage of red blood cells in whole blood is called:

Hematocrit

An increased number of the cells seen in the image, upon microscopic examination of urine is termed:

Hematuria

When red blood cells hemolyze, what is the molecule released that will give the plasma its reddish appearance?

Hemoglobin

In which disorder do neonates demonstrate the presence of Bart's hemoglobin that changes to beta chain tetramers in adults?

Hemoglobin H disease

A major advantage of NGS over previous sequencing technologies such as Sanger sequencing is what?

Higher Throughput

Large, one-celled, smooth-to-tuberculate macroconidia, and smooth or echinulate microconidia are typical of mycelial phase growth of which organism?

Histoplasma capsulatum

Identify the antinuclear antibody (ANA) staining pattern in the image (a) and interpret what related disorder can be associated with this pattern?

Homogeneous or Diffused staining pattern; Systemic Lupus Erythematosus

A completely sickled cell (drepanocyte) is most commonly seen in which of these conditions?

Homozygous HbSS

The Major Histocompatibility Complex (MHC) I genes encode for:

Human Leukocyte Antigens (HLA)-A,B,C

Identify the urine sediment element indicated by the arrow in the illustration:

Hyaline cast

This suspicious form was found in a stool specimen and measures 54 µm by 32 µm. What is the identification?

Hymenolepis egg

Nephrotic syndrome is associated with all of the following clinical findings, except:

Hyperproteinemia

Which blood group is most frequently associated with Cold Agglutinin Disease (CAD)?

I

Rocky Mountain spotted fever (RMSF), caused by Rickettsia rickettsii, is MOST reliably diagnosed by which laboratory technique?

IFA testing

Examples of cytokines originating from activated T-cells include:

IL-2, IL-3

The most frequently encountered immunoglobulin demonstrated in patients with Multiple Myeloma is:

IgG

Which of the following antibody types is the predominant serum antibody seen in the secondary immune response:

IgG

The most significant blood group antibodies belong to which immunoglobulin classes?

IgG and IgM

Coombs control check cells used to verify negative reactions at the anti-globulin (IAT) phase are coated with:

IgG antibodies

What characteristic differentiates a secondary immune response from a primary immune response?

IgG is the predominant antibody class produced in the secondary immune response compared to the primary immune response.

A 27-year-old female graduate student recently returned from South America, where she completed a nature study of the rainforest and spent several months "living off the land." She consulted with her physician for treatment of a sinus infection, which she thought was possibly responsible for several recent bouts of diarrhea. After questioning the patient, the doctor decided to collect a stool specimen for culture and parasitic examination. The stool culture was reported as "no enteric pathogens isolated." This suspicious form was seen on both wet preparations and a permanent stain. It measures 17 µm. The identity of this form is most likely:

Iodamoeba butschlii

Amoebae stained with this substance may be readily distinguished because it enhances nuclear and structural detail.

Iodine

Which disease listed below is the result of a mitochondrial disorder?

Kearns-Sayer

The McLeod phenotype is associated with which of the following antigen systems?

Kell

A gram-positive bacillus grew as a diffusely beta-hemolytic colony from a newborn. It was catalase-positive and had tumbling motility on a hanging drop preparation. The image shows how the organism grew in the motility medium. What is the most likely diagnosis?

Listeria monocytogenes

Illustrated in this photograph to the right is a sulfide indole motility (SIM) tube (left) and an esculin hydrolysis slant (right) after inoculation with an unknown gram-positive bacillus and incubation at 30°C for 24 hours. Based on the reactions observed, the most likely identification is:

Listeria monocytogenes

Which organism matches the reactions shown? Left tube: Motility agar (note subsurface flare shown by arrows) Middle tube: Esculin hydrolysis (+) Right tube: Voges-Proskauer(VP) (+)

Listeria monocytogenes

All of the following are sites of active hematopoiesis in the adult EXCEPT:

Liver

Acute transfusion reactions are divided into categories based on all of the following symptoms EXCEPT:

Liver failure

Which of the following is a TRUE statement when performing maintenance or repair tasks on electrical equipment?

Lockout is the physical placement of a device on the breaker or the placement of the plug into a canister to prevent its use.

A 32-year-old teacher is seeing her internist for her yearly physical. Because her great-grandmother suffered from cardiac disease, she is concerned about her own risk for cardiac disease. She is not a smoker nor hypertensive and is not overweight. She is physically active and maintains a good diet and nutrition. Her physician orders a lipid panel and hs-CRP to evaluate her cardiac disease risk. TestPatient ResultReference RangeCholesterol189 mg/dL< 200 mg/dLTriglyceride140 mg/dL< 150 mg/dLLDL- cholesterol98 mg/dL< 100 mg/dL (optimal)HDL-cholesterol60 mg/dL> 59 mg/dL (optimal)hs-CRP0.9 mg/dL< 1 mg/dL (low risk) Based on age, history, and laboratory results, what is her risk of cardiac disease at this point?

Low risk

The hematology laboratory is experiencing instrument downtime. The laboratory has a backup method for hemoglobin, and hematocrit testing that is being used until the instrument is repaired. The hematologist calls and asks if you can provide any of the red blood cell indices. Which of the indices, if any, can you provide for the hematologist?

MCHC only

A cause of FALSE-POSITIVE result in the rapid plasma reagin (RPR) test for syphilis is:

Malaria

Which of the following viruses or diseases is screened for in blood donors only using questions asked during the donor screening and selection process

Malaria

A patient is seen at his physician's office for scaling, flat lesions on his back that are irregular with white blotches of discoloration (top image). The bottom image shown is of a tease mount taken from the skin and stained with Periodic Acid Schiff, which reveals narrow fungal hyphae (arrows) along with 3 - 5 µm in diameter, irregular sized spherical budding yeast cells, a presentation often referred to as "spaghetti and meatballs". Which fungi is the most likely cause of infection?

Malassezia furfur

The cellular immune response plays a role in whether a person exposed to Mycobacterium tuberculosis will develop tuberculosis disease (TB). Which of the following factors will make a person most at-risk of progressing from infection to active disease if exposed to M. tuberculosis?

Malnutrition

Which of the following is a valid reason for evaluating red cell morphology as part of the differential procedure?

May provide diagnostic information to the physician.

The presence of an increased number of hypersegmented neutrophils in the peripheral blood, as shown in this image, is an indication of which of the following conditions?

Megaloblastic anemia

All of the following statements are true regarding the hepatic system and drug metabolism, EXCEPT?

Metabolism of acetaminophen by the MFO system is altered by alcohol and makes it less toxic.

Which of the following is an example of a desirable characteristic of a Point of Care Testing (POCT) analyzer?

Method accuracy and comparability to the main laboratory method

The indicator(s) used in the pH test region of the chemical reagent strips for urine is/are:

Methyl red and bromthymol blue

The surface of a blood agar plate after 48 hours, incubated at 37oC, shows colonies with smooth, lemon yellow pigmentation. Note in the image is a Gram stain with relatively large Gram-positive cocci arranged in tetrads. Other distinguishing characteristics were the demonstration of susceptibility to bacitracin and resistance to furazolidone. Select the identification of this isolate

Micrococcus luteus

A peripheral smear demonstrates a population of red cells that are mostly 5 - 5.5 µm in diameter. The CBC data shows RDW of 13% (reference range 11.5-14.5%). Which of the following morphologies is consistent with these findings?

Microcytosis, low variation of cell volume

The calcofluor white stain can be used not only to help in the identification of fungi but can be used to visualize which of the following?

Microsporidia

An 18-year-old woman woke up in the morning to find that her left knee was hot, swollen, and very painful. She could not walk to work and a friend offered her a ride to the clinic. She had never had a swollen joint before. Physical exam revealed a tender knee joint which yielded purulent synovial fluid on aspiration. Gram-negative intracellular diplococci were seen on the Gram stain. She also had a cervical discharge that was cultured. The cause of her infection would grow best on:

Modified Thayer Martin (MTM) agar at 35º C with CO2

Each of the following characteristics is a prerequisite for a bacterial species to be classified within the family Enterobacteriaceae except:

Motility via peritrichous flagellae

The following diagram demonstrates two Serum Protein Electrophoresis (SPEP) patterns from different patients. The pattern on the left appears to be from a normal, healthy patient. The pattern on the right is from a patient with a particular disorder. What disorder is suggested by the pattern on the right?

Multiple Myeloma

You are collecting a blood specimen to be used for forensic (legal) alcohol testing. Which of the following must be done before you can start the specimen collection process?

Must inform the patient that the blood about to be collected is for alcohol testing.

Which of the following distinguishing characteristic is associated with Streptomyces species?

Musty basement odor

Illustrated in the photograph are 14-day-old colonies of a Mycobacterium species growing on Middlebrook 7H10 agar, with development of yellow pigmentation after exposure to environmental light. From the choices listed below, select the correct presumptive identification of this photochromogenic Mycobacterium species.

Mycobacerium marinum

llustrated in the photograph are 14-day-old colonies of a Mycobacterium species growing on Middlebrook 7H10 agar, with development of yellow pigmentation after exposure to environmental light. From the choices listed below, select the correct presumptive identification of this photochromogenic Mycobacterium species.

Mycobacerium marinum

The organism cultured on the Lowenstein-Jensen agar slant shown in the image to the right is most likely:

Mycobacteria

Several patients developed subcutaneous abscesses at the sites of injection of a preparation labeled "adrenal cortex injection" administered as part of a weight-loss regimen. Pale-staining, filamentous acid-fast bacilli were seen in the material aspirated from the lesions (the upper photomicrograph is representative) and the colonies shown in the lower photograph grew out after 3 days of incubation at 35°C in CO2. The most likely identification is:

Mycobacterium abscessus

Which Mycobacterium species will have a negative result for niacin, nitrate reduction, Tween hydrolysis (5 days), and iron uptake?

Mycobacterium bovis

An acid-fast bacillus recovered from a post-surgery wound of a young child had the following characteristics: Rapid growth (3-5 days) on blood and chocolate agar Nitrate positive Niacin negative Urease positive Catalase 68ºC positive Iron uptake positive Which of the following Mycobacterium species does this represent?

Mycobacterium fortuitum

All of the following Mycobacterium species produce pigments, EXCEPT?

Mycobacterium fortuitum group

Which Mycobacterium species has the shortest growing time on agar media when recovered from clinical specimens?

Mycobacterium fortuitum/chelonae

Illustrated here are colonies growing on Middlebrook 7H10 agar before (lower) and after (upper) exposure to light. Colonies are intermediate between smooth and rough depending on the strain. Prior to light exposure, the colonies are gray-white. Upon exposure to ambient light, visible yellow pigment production is observed as characteristic of one of the "photochromogens", as illustrated in the top composite colony photograph. The acid-fast stain reveals slender acid-fast bacilli with cross banding. Characteristic of this isolate are positive reactions for several biochemical reactions - nitrate reduction, Tween 80 hydrolysis, catalase, and urease. With these observations, select the species name of this isolate.

Mycobacterium kansasii

llustrated in the top photograph are dry, chalky, heaped colonies growing on Middlebrook agar with a distinctive yellow-orange pigmentation. They were recovered after 5 days of incubation from a subcutaneous lesion of the foot from a South American field laborer. Illustrated in the acid-fast stain in the lower photomicrograph are slender, branching acid-fast bacilli. Casein and tyrosine were hydrolyzed. With these observations and reactions, select the species identification of this isolate.

Nocardia brasiliensis

The following aerobic, branching, filamentous gram-positive species is weakly acid fast and known to cause mycetomas and lung infections in immunocompromised individuals:

Nocardia species

All of the following are characteristic of Burkholderia pseudomallei EXCEPT?

Non-motile

Which of these descriptions correlates best with the red cell distribution curve shown?

Normal red cell distribution

Various methods have been employed for the detection of Clostridioides difficile disease, which method is the new gold standard for detection?

Nucleic acid amplification tests (NAATs)

The following steps were used in a procedure to identify M. kansasii: 1 - Extraction 2 - Denaturization 3 - Hybridization 4 - Hydrolysis of excess reagent 5 - Detection of acridinium ester Which of the following methods has been described?

Nucleic acid probes

Which of the following statements is true regarding hyposegmented neutrophils?

Nucleus may be round, peanut-shaped or bilobed.

Which EBV markers would be MOST likely positive for an individual who had infectious mononucleosis nine years ago?

Only anti-VCA (IgG) and anti-Epstein-Barr nuclear antigen (EBNA) are positive.

Two biologically functional characteristics of immunoglobin G (IgG) are:

Opsonization of antigens for phagocytosis by macrophages and neutrophils; neonatal immunity by transfer of maternal antibodies across the placenta and gut of the fetus.

What is the nucleated cell present in the following image?

Osteoblast

The Sudan III stain is used to confirm the presence of:

Oval fat bodies

The measurement of total glycosylated hemoglobin A1c is an effective means of assessing the average blood glucose levels:

Over the past 2-3 months

Which of the following might cause a false positive indirect antiglobulin test (IAT)?

Over-centrifugation

All of the following are characteristics of Brucella are EXCEPT:

Oxidase negative

The bacterial species producing the set of reactions seen in the oxidative/fermentative (OF) glucose tubes illustrated in this photograph at the right is a (an):

Oxidizer

The adult flukes related to this 100 µm ovum typically inhabit the lung parenchyma; ova are most commonly found in sputum specimens. Diagnostic features for the ovum include a slightly thickened shell with a prominent shouldered operculum (as seen at the left lower end), a developing larva within the shell that is retracting from the inner margin leaving a clear space. Infections with this fluke are endemic in the orient, West Africa, South and Central America, and may be observed in patients who have traveled to one of these regions. Human infection occurs following ingestion of poorly cooked crab meat or crayfish contaminated with metacercariae. Intermittent chest pain and cough with occasional spitting up of blood (hemoptysis) may be experienced after infection. From the choices below, select the trematode species related to this ovum.

Paragonimus westermani

This large ovum (100 µm), as seen in the high-power photomicrograph, may be most commonly recovered in stool specimens from people presenting with low grade abdominal pain and diarrhea. Ova may also be recovered from sputum specimens as the adult worms may reside within lung pseudocysts, and during periods of coughing, ova may be released into the bronchi and observed in sputum. Based on the distinct features of the ova as presented in the photomicrograph, what is the presumptive identification?

Paragonimus westermani

Which type of healthcare occupational exposure has the greatest risk of HIV transmission?

Percutaneous injury

A tech is working on a urine culture and finds the following isolates: Escherichia coli: >100,000 CFU/mL Staphylococcus epidermidis: 5,000 CFU/mL How should the tech continue with the culture workup?

Perform sensitivity on E. coli and list the S. epidermidis as a contaminant

When a few small, purple inclusions are found in erythrocytes, they can be confirmed as containing iron by:

Performing a Prussian blue stain

Which of the following conditions would produce the results listed below in an anemic patient? MCV = 115 fL MCH = 30 pg MCHC = 34 %

Pernicious anemia

The study of how variations in the human genome affect a given individual's response to medications is called?

Pharmacogenomics (Pharmacogenetics)

Of the following anticonvulsant medications, which one does not rely solely on the liver to metabolize it for excretion?

Phenobarbital

Of the following, which has the longest half-life?

Phenobarbital

Illustrated in this image is the fruiting head of one of the fungal agents of chromomycosis. Note the long urn-shaped phialides tapered at the ends and topped with a distinctive flat, saucer shaped cap. Which of the following choices represents the type of sporulation illustrated in this image?

Phialophora sporulation

All of the following are considered opportunistic pathogens EXCEPT:

Phialophora verrucosa

In observing the slow-growing, 7-day-old dark brown colonies growing on Sabouraud Dextrose with Brain Heart Infusion (SABHI) agar, note that the black pigment extends to the reverse of the colony. This provides a presumptive indication of one of the fungi involved in chromoblastomycosis. The species identification of this fungus can be made by observing the distinctive fruiting heads illustrated in the bottom image. Select the identification of the fungal species.

Phialophora verrucosa

This organism is a slow-grower on Sabouraud Dextrose Brain Heart Infusion (SABHI) agar. It was recovered from a chronic recurrent cyst of the skin of the lower leg. The brown hair-like surface pigmentation appears brown-black when observed on the reverse of the colony. The identification is made by the microscopic appearance of fruiting heads. The image shows hyphae that are dark with flask-shaped phialides which form on the tips of the conidiophores. What is the name of this fungus?

Phialophora verrucosa

Illustrated in the top image is a 4-day growth of a dark brown, entire colony with a narrow outer white zone. The surface mycelium is glabrous and finely granular. As this colony is not specific, the fungal species identification must be made on stained mounts prepared from the surface of the mycelium. In the lactophenol blue mount shown in the bottom photomicrograph is the characteristic sac-like structure, a pycnidium, within which are contained small spiral, hyaline, one-celled conidia. Select the name of the fungal species represented in these photographs.

Phoma species

An electrolyte panel (lytes, chem-4) consists of:

Potassium, Sodium, Chloride, Carbon dioxide (in the form of bicarbonate)

When operating a centrifuge, what happens if the centrifuge is not properly balanced?

Potential breakage of collection tubes

"After repeated analysis, the same result is achieved" describes which of the following quality terms?

Precision

A doctor is requesting an acid-fast culture on a stool specimen. How should the tech handle this request?

Prepare a direct smear from the sample and only digest the specimen for culture if the smear is positive

A tech in the laboratory is reading a Gram stain from a cerebrospinal fluid. After searching the entire length of the slide for a few minutes, the tech was unable to locate any material (cells or bacteria) on the slide. How should the tech proceed?

Prepare a new slide by using cytocentrifugation

What is another name used to designate a fully committed B-lymphocyte:

Plasma cell

The cell in this image is known as a MOTT cell. What is the condition in which these cells are associated?

Plasma cell myeloma

Donation of which apheresis blood product more than once every four weeks requires monitoring of total plasma protein and antibody levels?

Plasmapheresis

A 3-year-old child was brought into the hospital with complaints of recurring conjunctivitis along with "woody" growths on the mucous membranes. Which of the following deficiencies would occur given these symptoms?

Plasminogen

Which Plasmodium species tends to infect any red blood cell, regardless of the age of the cell?

Plasmodium falciparum

The organisms listed below are given with the most appropriate specimen for their recovery, EXCEPT:

Plasmodium falciparum - ocular secretions

This parasite was found on a Giemsa stained blood smear. The form shown in the image at the arrow is best described as:

Plasmodium ring form

Thick Giemsa-stained blood smears are primarily used to screen for the presence of:

Plasmodium species

A patient has a history of repeated spontaneous abortion. Coagulation studies reveal an elevated APTT, normal PT, normal platelet function, and normal thrombin time. Schistocytes were seen on the peripheral blood smear. Which test should be performed to determine if the patient has lupus anticoagulant?

Platelet neutralization test

Which one of the following blood components would be MOST appropriate for a 9-yr old girl who is suspected of having immune thrombocytopenic purpura (ITP)?

Platelets

Which is the best method for examination of synovial crystals?

Polarized light

Which of the following biochemical reactions is most useful in establishing a definitive identification of Staphylococcus aureus?

Positive tube coagulase test

A PYR reaction is shown in the attached image. Which combination of result interpretation and possible organism identification based on the result is correct?

Positive/Enterococcus sp

The presence of only slightly visible hemolysis will significantly increase the serum level of which of the following electrolytes?

Potassium

Which of the following electrolytes is most likely to be spuriously elevated in a hemolyzed specimen?

Potassium

The oxidation of hemoglobin to methemoglobin in acidic urine after several hours may cause which of the following color to produce?

Red to brown

Which of the following cells is indicative of Hodgkin's disease?

Reed-Sternberg (RS) cells

On a patient that has a large amount of glucose in their urine, what would be the expected specific gravity (SG) taken from a refractometer in comparison to the dipstick SG?

Refractometer SG would be higher than Dipstick SG.

All of the following are functions of cerebrospinal fluid, EXCEPT?

Regulate body metabolism.

Which of the following descriptions best describes the term absolute value as it would relate to leukocyte differential counts.

Relative % of each cell type multiplied by total white count

The descriptions below refer to the appearance of inclusions found in Wright-stained or supravital-stained peripheral blood smears. Which best describes basophilic stippling?

Relatively evenly distributed fine or coarse granules throughout the red cell

A protected brush specimen was received into the microbiology laboratory from an ICU patient and cultured using a 0.001-mL loop. The culture grew 3 colonies of Pseudomonas aeruginosa. Which of the following is the correct way to report out the organism?

Report out 3,000 CFU/mL of Pseudomonas aeruginosa

Examine the following test results and identify what action you would take. CAMP: Arrowhead appearance Bacitracin disk: 6mm zone of inhibition PYR: Negative Bile Esculin: Tan appearance, no darkening

Report the isolate as Streptococcus agalactiae

Gram positive cocci isolated from a catheterized urine culture on a 76-year-old male gave the following reactions: Blood agar- creamy, white, opaque colonies with are not hemolytic Catalase- positive Slide coagulase- negative Tube coagulase- negative Novobiocin- susceptible The next action that the medical laboratory professional should take is to:

Report the isolate as coagulase negative staphylococcus

This Gram-positive coccus is catalase positive. The tests shown are a tube coagulase test inoculated with the isolate, and an MRSA screen plate containing 4% NaCl and 6 µg/mL of oxacillin. A 0.5 McFarland standardized inoculum of the isolate is inoculated to the plate. What step should you take next?

Report the isolate as methicillin-resistant Staphylococcus aureus.

A semen specimen was collected three hours before it was brought to the laboratory for examination. What course of action should be taken?

Report the specimen as compromised on the final report.

You are working in a hematology laboratory, and the machine flags an outlier result for a 51-year-old male. The blood was collected in an EDTA tube for a CBC. The hematocrit of the specimen is high at 60%. Which of the following would you do?

Report these results.

Smooth gray-white, convex, glistening non-hemolytic colonies were observed growing on the surface of anaerobic blood agar after 48 hours. This isolate was obtained from a nasal swab specimen. Short Gram positive bacilli in short chains and in diphtheroidal-like clusters were observed on Gram stain. Spot test results included a positive catalase and a negative indole reaction. This organism was presumptively identified as Cutibacterium (Propionibacterium) acnes. With this presumptive identification, select from the multiple choices what the next step should be.

Report with added comment: "probable contaminant"

The photograph here is of a peripheral smear sent for hematologic review. No clinical information for the patient was sent with the slide. What is the first course of action that the reviewer should take to assist him/her in interpreting the findings on this blood smear?

Request additional medical information

Gram stain of a spinal fluid revealed pleomorphic, gram-negative rods. Colonies that appeared on chocolate agar after 24 hours of incubation were gray, opaque, and medium in size. A musty odor was noted when the plate was opened. Gram stain of the colonies on chocolate agar showed small, gram-negative coccobacilli. No growth was seen on sheep blood or MacConkey agar. The next step taken by the technologist to identify this isolate would be to show that the organism:

Requires X and V factors

Parasitized animals that may serve as a source of infection for humans are called:

Reservoir hosts

The mutation in the ß-globin allele leading to sickle cell disease:

Results in an aminoacid substitution of Glutamic acid with Valine.

Which of the following statements are true for the hematopoietic cords?

Reticular cells (adventitial cells) provide support for developing hematopoietic cells within the hematopoietic cords

Which of the following Enterobacteriaceae produces H2S?

Salmonella species

Examine the KIA tube on the right in the image provided and determine which of the following organisms produces the biochemical reaction seen.

Salmonella typhi

Illustrated in the upper image is a cercaria capable of penetrating the skin of a human swimming in contaminated water. These cercaria then enter the circulation, and mature in the portal vascular system where they develop into adult flukes. Distinctively the female flukes occupy the gynecophoral canal of the male fluke, as illustrated in the lower image. From the list of trematode genera, select the genus that represents these stages of this life cycle.

Schistosoma

Which white blood cell is found in the largest numbers in the peripheral blood of a normal adult?

Segmented neutrophil

The ONPG (ortho-nitrophenyl-Beta-D-galactopyranoside) test is used to:

Separate late lactose fermenters from lactose-negative bacteria based on the production of beta-galactosidase.

The positive phenylalanine deaminase reaction seen in the left tube (green pigmentation in the slant compared to the negative control on the right) may be produced by each of the following bacterial species EXCEPT:

Serratia marcescens

All of the following Enterobacteriaceae below are appropriately matched with the characteristic listed, EXCEPT:

Serratia marscens - Voges-Proskauer negative

All of the following bacterial species produces a yellow-pigmented colony when growing on 5% sheep blood agar EXCEPT:

Serratia rubidaea

What is the anticoagulant most commonly used for routine coagulation testing?

Sodium citrate

Which one of the following is a detergent added to gel media that enhances the separation of solutes during electrophoresis?

Sodium dodecyl sulfate

An electrolyte panel is composed of which of the following tests?

Sodium, potassium, chloride, CO2

Identify the anti-nuclear antibody (ANA) pattern in this microscopic image.

Speckled

This antinuclear antibody (ANA) pattern is characterized by granular staining in the nuclei of the interphase cells (a). There is also an absence of staining in the chromosomal area of the metaphase mitotic cells (b). The slide is viewed using fluorescent microscopy. Which pattern is this?

Speckled

Varied red blood cell morphologies due to pitting, culling, filtering, and the persistence of abnormal red blood cells in the peripheral blood is most likely due to what?

Splenectomy

Often recovered from puncture wounds of the fingers, wrists and arms, illustrated in the top photograph is the 4 day growth on SABHI agar after incubation at 30o C. The colony is circumscribed, yellow, rough and wrinkled centrally, surrounded by narrow smooth, white circular growth with a distinct brown-black outer border. The identification can be made by observing the lactophenol blue stained fruiting heads in the bottom photomicrograph. From these observations, select the name of this fungal isolate.

Sporothrix schenckii

A tech is reviewing a Gram stain from a positive blood culture bottle. The background on the stain shows pink debris and the tech thinks that there is also Gram-negative bacilli but is having difficulty differentiating the artifact and possible organisms. What could the tech do next to help determine if bacteria are present in the blood culture bottle?

Stain with Acridine Orange

An infective agent of skin wound infections and an agent of toxic shock syndrome, the large, entire, golden-yellow, smooth convex colonies on blood agar were recovered from a swab specimen after 48 hours incubation at 37o C. In the image to the right, clusters of gram-positive cocci were observed microscopically in a gram stain prepared from one of the colonies. The coagulase reaction was positive. With these observations, select the name of this isolate from the choices given.

Staphylococcus aureus

Purulent material is obtained from a carbuncle and submitted for bacterial culture. Many Gram-positive cocci and white blood cells were seen on a direct smear (top right image). Colonies on blood agar are, smooth and gold/yellow surrounded by a narrow zone of beta hemolysis (top left image). Coagulase tests are shown in the bottom images. The slide coagulase test is negative, but the tube coagulase test is positive. The probable presumptive identification is:

Staphylococcus aureus

The slide coagulase test is a rapid method for identifying which of the following organisms?

Staphylococcus aureus

Which of the following Staphylococcus species will produce yellow colonies on mannitol salt agar?

Staphylococcus aureus

Which of the following gastrointestinal pathogens has an incubation period of 1-8 hours?

Staphylococcus aureus

Which of the following is LEAST likely to stimulate the production of reactive lymphocytes?

Staphylococcus aureus

A Staphylococcus species recovered from blood culture was found to produce acid from sucrose and maltose and showed alkaline phosphatase activity. The Staphylococcus was also coagulase-negative. The same organism was also found in a culture from the central line tip. The most likely identification is:

Staphylococcus epidermidis

Each of the bacterial species listed below may be associated with culture-negative endocarditis EXCEPT:

Staphylococcus epidermidis

This is a catalase-positive, coagulase-negative, gram-positive coccus isolated from a urine specimen from a 20-year-old female college student. The image shown is a Mueller Hinton plate streaked with a 0.5 MacFarland standardized inoculum and a 5 microgram disk of novobiocin after overnight incubation. What is the identification of the isolate?

Staphylococcus saprophyticus

The entire, smooth, golden yellow-pigmented non-hemolytic colonies on the surface of the blood agar plate seen in the upper image have been recovered in hospital acquired infections from patients with cystic fibrosis. A Gram stain prepared from one of the colonies as shown in the lower image, reveal long, narrow gram-negative bacilli arranged singly. Polar flagella would be seen in flagellar stains. All carbohydrate reactions are negative except for the oxidative utilization of glucose and maltose and the organism tested negative for oxidase. Positive biochemical reaction for lysine assists in making a definitive identification. Based on these observations, select the identification of this isolate.

Stenotrophomonas maltophilia

What are the cells that are indicated by the arrows in this slide?

Stomatocytes

A 67-year-old man was seen in the emergency room complaining of cough, fever, and piercing right posterior chest pain. X-ray of the chest revealed consolidation of the right middle lobe of the lung. Sputum culture grew the bacterial species shown in the upper photograph. The lower photomicrograph illustrates a gram-stain of the sputum specimen. The most likely cause of the pneumonia is:

Streptococcus pneumoniae

Small, gray, alpha hemolytic colonies with indented centers were isolated on sheep blood agar from a peritoneal fluid. The following biochemicals were performed: Gram Stain: Gram positive lancet-shaped cocci in chains and pairs Catalase: Negative Optochin susceptibility: Sensitive >14 mmIdentify the organism from the choices provided.

Streptococcus pneumoniae

The optochin (ethylhydrocupreine hydrochloride) disk is most often used for the identification of which organism?

Streptococcus pneumoniae

While many isolates of this organism remain susceptible to penicillin, resistance is increasing in some areas due to the organism's altered penicillin-binding proteins. In such cases, the recommended treatment is erythromycin or chloramphenicol. Which of the following organisms does this describe?

Streptococcus pneumoniae

A stool specimen has been left at room temperature overnight. The next day, many motile larvae with a short buccal cavity are seen upon microscopic exam, but there are no ova observed. What is the most likely identification and life stage of this parasite?

Strongyloides stercoralis - rhabditiform larvae

All of the following are quantitative methods for the determination of albumin EXCEPT?

Sulfosalicyclic acid precipitation

Which acid is most frequently used to precipitate protein for confirmatory urinalysis tests?

Sulfosalicylic

What test can be used as a confirmatory test for proteinuria?

Sulfosalicylic acid test

The arrows in the image point to Heinz bodies in erythrocytes. Which of the following stains was used to confirm the presence of these inclusions?

Supravital stain

Given the following laboratory results, what is the most likely diagnosis? HBsAg: Negative Anti-HBc: Negative Anti-HBs: Negative

Susceptible to Hepatitis B infection

all of the following pose a significant risk of transmitting bloodborne pathogens, EXCEPT?

Sweat

Which of the following represents the best match of cell type with function?

T lymphocytes - cell mediated immunity

Identify this form that was found in a stool concentrate, measuring 33 µm in diameter

Taenia species egg

All of the following would be effective methods to reduce the risk of becoming infected with the H1N1 virus, EXCEPT?

Take an aspirin every day.

The red cells in this illustration exhibit which of the following abnormal erythrocyte shapes:

Teardrop cells

What is the predominant abnormal erythrocyte morphology associated with idiopathic myelofibrosis?

Teardrop cells

The principal amount of testosterone in males is produced by which endocrine gland?

Testes

What is an appropriate use for molecular (DNA) tests for mutations of the HFE gene, the gene found in the majority of patients diagnosed with hereditary hemochromatosis (HH)?

Testing family members of persons with HH

When a Specked Nuclear Antibody (ANA) pattern is observed, what follow-up test for antibodies related to Systemic Lupus Erythematosus (SLE) is not beneficial?

Testing for antibodies to U1RNP+ and dcSSc

A Bombay individual's blood specimen can be differentiated from a blood specimen of a group O person by which of the following?

Testing with anti-H lectin (Ulex europaeus)

What must be true for the antiglobulin phase of the serologic crossmatch to be omitted (i.e., immediate spin crossmatch is done)?

The antibody screen must be negative and there is not history of detection of unexpected antibodies.

All of the following statements concerning Rickettsia species are true, EXCEPT:

They are cultured in many hospital laboratories.

Which of the following describes the Westgard multirule 22s?

Two consecutive data points fall outside +2SD or fall outside -2SD.

A patient suspected of a urinary tract infection has a negative nitrite test, but bacteria is present upon microscopic examination. What may have caused this discrepant result?

The bacteria present is not a nitrate-reducer.

You are performing a manual antibody titer in a clinical laboratory. You have made serial dilutions of a patient's plasma sample and you are looking for anti-streptolysin O antibodies. The first tube contains pure patient plasma. The second tube contains a 1:2 dilution of the first tube. The third tube contains a 1:2 dilution of the second tube and so on. The patient's antibodies no longer give visible reaction in tube 5, indicating their titer is tube 4 (or the last tube where there is a positive result). What is the concentration of antibodies in tube 4 compared to the concentration in tube 1?

The concentration of antibodies in tube 4 is 1/8 the concentration of antibodies in tube 1.

A patient who previously had a hs-CRP of 4 mg/L was put on a statin and daily aspirin medication. She returns to the laboratory a month later, and her repeat hs-CRP is 2.3 mg/L. Which of the following is the most likely reason for the decrease in the test value?

The medications lowered the hs-CRP value.

You are analyzing a patient sample in the hematology laboratory when the instrument you use for automatic CBC flags your specimen for having a low platelet count. Upon performing a visual differential of the slide, you notice that there are indeed very few platelets on the slide. You also notice numerous, fragmented red blood cells. Which of the following choices could be associated with the low platelet count and fragmented red blood cells found in this case?

Thrombotic Thrombocytopenic Purpura (TTP)

Aspirin ingestion prevents the synthesis of this signaling molecule in the platelet?

Thromboxane A2

What is the role of a sentinel laboratory within the Laboratory Response Network (LRN)?

To rule-out critical biological agents or to refer them to a Laboratory Response Network (LRN) reference laboratory

Which of the following bone marrow sampling methods would be advisable if the quantity is inadequate for an aspirate smear?

Touch imprint

What function in the body does GGT have?

Transfer of gamma-glutamyl peptides amino acids, water, and other peptides

Total iron-binding capacity measures the serum iron transporting capacity of:

Transferrin

Which of the following is currently considered to be a good test for screening persons for hereditary hemochromatosis (HH) due to its sensitivity and specificity for iron overload?

Transferrin saturation

Which of the following transfusion reactions can a diagnosis be more firmly established by evaluating B-type natriuretic peptide (BNP) levels before and after transfusion?

Transfusion Associated Circulatory Overload (TACO)

Which protein is also known as prealbumin because it migrates before albumin in the customary electrophoresis of serum or plasma proteins?

Transthyretin

llustrated in this photomicrograph is an H & E - stained section of skeletal muscle illustrating the larval form of an invading parasite (arrow). Humans become infected by ingesting raw or poorly cooked infected animal meat. An epidemic occurred in the 1800s during the North Pole expedition when explorers were eating the raw and undercooked meat of infected indigenous polar bears that they had killed for food. What is the species shown?

Trichinella spiralis

Illustrated in this photomicrograph is a 10 µm in diameter, trophozoite observed in a trichrome-stained vaginal smear from a 39-year-old female experiencing a frothy vaginal discharge along with vaginal discomfort and pain on urination. What is the name of the invasive parasite?

Trichomonas vaginalis

A 31-year-old male missionary worker who had recently returned from Africa where he helped a small rural community update their sanitation practices, presented to his physician complaining of weakness, recent weight loss, abdominal pain, and diarrhea that was often bloody. A battery of tests was ordered including a complete blood count (CBC) and stool for parasite examination. The CBC revealed eosinophilia and anemia. This suspicious form that measured 52 µm by 27 µm was seen in the stool concentrate. What is the most likely identification of this organism?

Trichuris trichiura

A 4-year-old female from South Carolina suffering from malaise, bloody diarrhea, and abdominal pain, was rushed to the emergency room. Physical examination revealed a rectal prolapse. A stool specimen was submitted for parasitic examination and this suspicious form, measuring 45 µm by 20 µm, was seen. What is the identity of this organism?

Trichuris trichiura

Whipworm infection is typically diagnosed by the microscopic observation of adult worms or barrel-shaped eggs with polar plugs in stool specimens. What organism is known as the whipworm?

Trichuris trichiura

Normal crystals that can be found in urine include:

Triple phosphate

What is the identification of these crystals seen in urine with an alkaline pH?

Triple phosphate

The colony shown in the top image developed on Sabouraud's agar in 3 days when incubated at 30° C. When recovered in laboratory cultures, the source is either from the environment or a commensal contaminant. The colony has a woolly brown, gray-black, and white circumferential appearance, with the gray-black pigment being observed from the reverse side of the petri dish. From the microscopic appearance as illustrated in the bottom photograph, what is the most likely identification?

Ulocladium species

All of the following are laboratory centrifuges, EXCEPT?

Ultraviolet (UV) centrifuge

A 25-year-old female presented in the emergency room with an acute urethral discharge of two days duration. The image to the right shows the Gram-stained smear that was obtained. Many polymorphonuclear leukocytes and intracellular and extracellular gram-negative diplococci were observed. Based on the clinical history and the Gram stain observation, what diagnosis can be made?.

Unable to determine the cause of infection

This microscopic field is representative of other fields that were observed on a Gram-stained smear. Which of the following describes the quality of the smear?

Under-decolorized; Poor Quality

To establish an identification of Trichomonas vaginalis from saline wet mounts of vaginal specimens, the organism must be motile. Which of the following is used to identify Trichomonas vaginalis?

Undulating membrane

Which of the following is the most commonly encountered presenting sign of a delayed hemolytic transfusion reaction (DHTR)?

Unexplained decrease in hemoglobin or hematocrit

Which analyte should be increased in the presence of gout?

Uric acid

Select from the multiple choices the specimen that is inadequate for the recovery of anaerobes and should be summarily rejected upon receipt in the microbiology laboratory.

Uterine cervical swab in Cary-Blair transport medium

What cellular characteristic of red blood cells is described by the term poikilocytosis?

Variability in shape

The veins in the antecubital area of the forearm are the primary site for blood collection. If the phlebotomist is unable to locate a vein in the antecubital area, what is the second-best site location for collecting blood?

Veins in the back of the hand

Representative assays in the category of treponemal methods include the listed assays except:

Venereal Disease Research Laboratory Test

Gram stains are performed on positive blood culture bottles. Each of the following responses correctly matches the organism with its description EXCEPT:

Very tiny, pale staining, gram-negative coccobacillus often arranging in pairs = Fusobacterium species

A patient presents to the emergency room with severe headaches, nausea, and vomiting. CSF chemistry results are as follows: Protein - 74 mg/dL (15-45 mg/dL) Glucose - 53 mg/dL (40-85 mg/dL) Based on the patient's complaints and the above laboratory results what is the most probable condition?

Viral Meningitis

VKORC1 and CYP2CP genotypes are important for determining the dosage of which drug?

Warfarin

Identify the urine sediment elements shown by the arrow:

Waxy cast

What is the MOST likely cause of the ABO discrepancy when the following results were obtained from a first-time 29-year old, blood donor? Forward Group Anti-A = Negative Anti-B= Negative Reverse Group A1 Cells = Negative B Cells = 3+

Weak subgroup of A

An 18-year-old immigrant from the Philippines presented to the local clinic shortly after relocating to the United States complaining of fever and chills. Physical examination of the patient revealed enlarged lymph nodes. Blood was drawn and submitted for culture and parasitic examination. The culture was negative. This suspicious form was seen on the Giemsa-stained blood smear. It measures 225 µm in length. This patient is most likely infected with:

Wuchereria bancrofti

A stool culture from a hospitalized patient produces the following characteristics: Gram-negative bacillusUrease positiveWeakly fermentativeTSI slant is A/A These reactions suggest that the organism is most likely:

Yersinia enterocolitica

Below are the laboratory results on a 20-year-old overweight male. His waist circumference is 41 inches and his blood pressure is 105/70. Fasting Blood Glucose: 130 mg/dLTriglycerides: 190 mg/dLHDL: 50 mg/dL The National Heart, Lung, and Blood Institute (NHLBI) and American Heart Association (AHA) have created a set of parameters to define the presence of the metabolic syndrome. Does this patient meet the criteria for metabolic syndrome diagnosis?

Yes, he meets the NHLBI and AHA criteria for metabolic syndrome.

The gene loci for the alpha-globin chains are adjacent to the locus for which other globin chain?

Zeta

Positive HPV PCRs typically include genotyping or a reflex to genotyping to identify higher-risk genotypes for cancer development. Which class of HPV contains the most high-risk genotypes for cervical cancer?

a group (16 and 18)

In which of the following conditions would a technologist NOT expect to see the misshaped "spiked" erythrocytes included in the photograph?

abetalipoproteinemia

In the Westgard system for evaluating control values, all of the following are rules for rejecting the run, EXCEPT?

12s

A phlebotomist who is performing venipuncture on a small delicate hand vein should choose a needle with a gauge number of:

23

A 200 mg/dL solution was diluted 1:10. This diluted solution was then additionally diluted 1:5. What is the concentration of the final solution?

4 mg/dL

According to Westgard and CLIA, how many specimens should be run by each method on the same day for 8 to 20 days to compare a test method with a comparative method?

40-100

A potential apheresis platelet donor must be deferred for how long following the use of aspirin?

48 hours

A technologist has 500 µL of serum and needs to make a 1/2 dilution for a glucose determination. How should he/she do it?

500 µL of serum plus 0.5 mL of saline

Using the formula for creatinine clearance, calculate the clearance based on the values given below: Urine creatinine: 100 mg/dL Plasma creatinine: 1.9 mg/dL Urine volume: 1375 mL/24hrs Surface Area: 1.2 m2 Round to 1 decimal place.

72 mL/min/m2

Streptococcus pneumoniae can BEST be differentiated from the viridans group of streptococci by which characteristic?

Bile solubility

All cells can synthesize heme; what two organs are the main sites of hemoglobin synthesis for the body?

Bone marrow and liver

Ionized calcium is currently most commonly measured using which of the following method?

Calcium ion selective electrodes

Which of the following serological tests would be used for the diagnosis of Q-fever?

EIA or indirect immunoflourescence

Corneal tissue transplantation has an extremely high success rate for a variety of reasons, including all of the following with the exception of:

Eccentrically place grafts

The term "hydatid sand" is used in reference to which of the following tapeworms?

Echinococcus species

The D-dimer is useful in detecting activity in which aspect of hemostasis?

Fibrinolysis

The colonies illustrated in the top photograph are 48-hour-old colonies growing on the surface of blood agar that had been incubated anaerobically. The colonies are small, gray-white, and convex with irregular borders and internal flecking. Long, slender fusiform Gram-negative bacilli with tapered ends are seen in the Gram stain as observed in the bottom photomicrograph. The indole reaction is strongly positive and glucose fermentation is variable; other biochemical reactions are negative except for H2S production by some strains. This isolate is commensal in the upper respiratory tract and has been associated with hospital-acquired aspiration pneumonia, lung abscesses, and empyema in hospitalized patients. Based on these observations, select from the multiple choices the name of this isolate.

Fusobacterium nucleatum

The parents' blood types are AB and O. What could be the only possible blood type(s) of their children?

Group A or B only

A ß-hemolytic isolate on SBA revealed a Gram-positive coccus arranged in chains. The isolate was catalase-negative, PYR negative, bacitracin resistant, and positive for CAMP and hippurate hydrolysis. What is the presumptive identification of this isolate?

Group B streptococci

Which of the following hazards is associated with the blue diamond in the NFPA hazard label shown below?

Health hazard: Extreme danger

A new tech has a presumptive Salmonella species via biochemical identification on a stool culture. To confirm, serological testing is performed. The serological testing is negative and the quality control which was run simultaneously passed. What should the tech do next?

Heat a suspension of the organism and then repeat the serological testing

B lymphocytes and T lymphocytes are derived from:

Hematopoietic stem cells

Each of the following tests is helpful in the species identification of Staphylococcus aureus EXCEPT:

Modified dimethyl sulfoxide (DMSO) test

Which of the following has been compared to a hockey puck when discussing its morphology on chocolate agar?

Moraxella catarrhalis

The images show disease presentations associated with Mycobacterium species infections. Which organism correlates to the disease presentation seen in the lower right image?

Mycobacerium scrofulaceum

What is the MOST likely identification of an acid-fast bacillus that demonstrates the following characteristics? Slow growth Cream to tan colored colonies when grown in the dark Development of yellow pigment upon exposure to light

Mycobacterium kansasii

The following is the MOST likely identity of an acid-fast bacilli that showed buff-colored, dry, heaped colonies; niacin, nitrate, and urease positive; and took 23 days at 35ºC to grow:

Mycobacterium tuberculosis

What type of white blood cell is seen MOST frequently in urine sediment with infections of the urinary system?

Neutrophil

Which dematiaceous fungi affects the joints causing septic arthritis?

Phialophora spp.

Critical values not reported or not reported in a timely manner is an example of a laboratory error made during which phase of laboratory testing?

Postanalytic

The primary purpose of neutrophil granules is to:The primary purpose of neutrophil granules is to:

Provide microbicidal action

Illustrated in the top photograph are colonies on blood agar after 48-hour growth at 37° C incubation. The colonies are non-hemolytic, small, entire, smooth, and gray-white. Small Gram-positive cocci arranged in short chains are seen on Gram stain as illustrated in the lower photomicrograph. The key to the identification is the rapid production of esculin and not growing in 6.5% NaCl broth. What is the identification of this organism?

Streptococcus equinus

Jane Doe is a 30-year-old female who felt tired for several months, had pain in the joints of her fingers, and recently developed a dermatitis following exposure to the sun. The following test results were obtained on a blood sample drawn during the initial evaluation: Total Protein = 8.4 gm/dL (N = 6.0-8.0 gm/dL) ANA >1:2560; speckled pattern CRP = positive C3 = 40 mg/dL (N = 80-180 mg/dL) C4 = 5 mg/dL (N = 15-45 mg/dL) Based on the clinical and laboratory findings, what disease should be suspected?

Systemic Lupus Erythematosus (SLE)


Related study sets

The Humanities, Natural Sciences, and Social Sciences

View Set

APUSH | Chapter 12 An Age Of Reform, 1820-1840

View Set

Microeconomics Terms and Definitions

View Set

Chapter 8: Organizational Culture, Structure, and Design: Building Blocks of the Organization ~ MGMT 320 ~ DR. TAMELA D. FERGUSON ~ ULL ~ WinterSession2020

View Set

Apush prior test questions, 1st semester final

View Set

Race and Ethnicity, Prejudice and Discrimination

View Set

Neurology - 602 Post Midterm Information

View Set

Chapter 14 Chapter Homework-Medical Terminology- The Reproductive System

View Set